[obm-l] Re: [obm-l] Congruência(?)

2013-08-20 Por tôpico Bernardo Freitas Paulo da Costa
2013/8/20 marcone augusto araújo borges marconeborge...@hotmail.com
 Mostre que 46^47 + 48^47 divide 47^2
Você quer que um número gigantesco divida um número pequenininho? Ou é
ao contrário?

 Eu só consegui desenvolvendo (47 - 1)^47 + (47 + 1)^47.
 Como fazer por congruência?
Acho que dá pra fazer, mas no fim das contas é mais fácil expandir
como você pensou, porque para usar congruências você tem que fazer
tudo módulo 47^2... e daí as contas vão ficar feias... Note que o
pulo do gato é que C(47,p) é divisível por 47 para todo 0  p  47,
daí nem adianta muito você tentar simplificar 46^2 = (47 - 1)^2 ==
-2*47 + 1 (mod 47^2) e 48^2 == 2*47 + 1.

Abraços,
-- 
Bernardo Freitas Paulo da Costa

-- 
Esta mensagem foi verificada pelo sistema de antivírus e
 acredita-se estar livre de perigo.


=
Instruções para entrar na lista, sair da lista e usar a lista em
http://www.mat.puc-rio.br/~obmlistas/obm-l.html
=


Re: [obm-l] Russia-1997

2013-08-18 Por tôpico Bernardo Freitas Paulo da Costa
2013/8/18 marcone augusto araújo borges marconeborge...@hotmail.com
 Se a e b são números reais tais que a^3 - 3a^2 + 5a = 1 e b^3 - 3b^2 + 5b = 
 5,determine
 a + b.

Veja que P(a) = 1 e P(b) = 5, para um polinômio de terceiro grau, P(x).

Primeira coisa, veja que nesses casos há apenas uma raiz. (Talvez você
mostre até que P(x) = c tem uma única raiz real para qualquer real c)
Isso garante que o problema está bem-posto.

Se fizermos P(a) + P(b), temos dois termos com (a+b), um deles vindo
da fatoração de a^3 + b^3. Mas o termo quadrático não ajuda, e ainda
por cima resta a parte constante.

Assim, vamos eliminar o termo quadrático com uma substituição :
P(y+1) = (y+1)^3 - 3(y+1)^2 + 5(y+1)
=  (y^3 + 3y^2 + 3y + 1) - 3(y^2 + 2y + 1) + 5(y+1)
= y^3 + y(3 - 6 + 5) + (1 - 3 + 5)
= y^3 + 2y + 3

Sejam A = a-1 e B = b-1.

1 = P(a) = P(A+1) = A^3 + 2A + 3, e 5 = P(b) = P(B+1) = B^3 + 2B + 3, ou seja
-2 = A^3 + 2A
2 = B^3 + 2B

Que beleza!

0 = (A^3 + B^3) + 2(A+B) = (A+B)(A^2 - AB + B^2) + 2(A+B). Assim, ou
(A+B) = 0, ou

A^2 - AB + B^2 + 2 = 0. Mas essa equação do segundo grau (em A, por
exemplo) tem discriminante B^2 - 4(B^2 + 2) = -3B^2 - 8  0, ou seja,
não tem solução real.

Assim temos que ter A+B = 0, ou seja, a+b = 2.

Note que P(0) = 0, P(1) = 3 e P(2) = 6. Assim, a está entre 0 e 1, b
entre 1 e 2, e magicamente simétricos em relação a 1, da mesma forma
que 0 e 2.

Abraços,
--
Bernardo Freitas Paulo da Costa

-- 
Esta mensagem foi verificada pelo sistema de antivírus e
 acredita-se estar livre de perigo.


=
Instruções para entrar na lista, sair da lista e usar a lista em
http://www.mat.puc-rio.br/~obmlistas/obm-l.html
=


Re: [obm-l] Russia-1997

2013-08-18 Por tôpico Bernardo Freitas Paulo da Costa
2013/8/18 Jeferson Almir jefersonram...@gmail.com

 Cara dar para vc montar um polinomio bizu ai e ver que ele é Par certa vez um 
 amigo mostrou essa ideia nao estou conseguindo aqu se alguem conseguir 
 agradeço tambem desde de jái. .

  entao vc chama a-1=x e b-1=y

Então, quando você faz esta substituição, o polinômio P(x) = x^3 -
3x^2 + 5x vira Q(y) = x^3 + 2x + 3. Note que, a menos de uma constante
(o +3), este polinômio é ÍMPAR. E crescente, também. Assim, as
soluções de Q(y) = 3 + c e Q(z) = 3 - z são, necessariamente,
simétricas em relação a zero: Temos

y^3 + 2x = c
z^3 + 2z = -c

Como uma solução para a segunda equação é z = -y, e como há uma única
solução, este é o valor de z. Assim, y+z = 0. Desfazendo a
substituição, a soma é sempre 2 para a,b tais que P(a) = 3 - c, P(b) =
3 + c.

Abraços,
--
Bernardo Freitas Paulo da Costa

-- 
Esta mensagem foi verificada pelo sistema de antivírus e
 acredita-se estar livre de perigo.


=
Instruções para entrar na lista, sair da lista e usar a lista em
http://www.mat.puc-rio.br/~obmlistas/obm-l.html
=


Re: [obm-l] Ajuda num problema

2013-08-12 Por tôpico Bernardo Freitas Paulo da Costa
2013/8/12 Marcelo de Moura Costa mat.mo...@gmail.com
 Gostaria de que alguém me orientasse nesse problema, mas é para atender
 alunos da 9ºano, pois é do Colégio Naval.

Veja que(3 + 2*raiz(2)) e (7 + 5*raiz(2)) são ambos potências do mesmo
número (irracional).

--
Bernardo Freitas Paulo da Costa

-- 
Esta mensagem foi verificada pelo sistema de antivírus e
 acredita-se estar livre de perigo.


=
Instruções para entrar na lista, sair da lista e usar a lista em
http://www.mat.puc-rio.br/~obmlistas/obm-l.html
=


Re: [obm-l] off-topic charada

2013-07-30 Por tôpico Bernardo Freitas Paulo da Costa
2013/7/30 Hermann ilhadepaqu...@bol.com.br:
 por que motivo de cada dez livros de álgebra linear tenho dez sumários
 diferentes?

 estou errado em pensar assim?
Eu acho que você está exagerando um pouco. Muitos livros de AL fazem
Matrizes - Determinantes - Bases - Diagonalização. Muitos outros,
Espaços vetoriais - Transformações lineares - Matrizes. Há alguma
variação dentro disso, mas será que é tanto assim?

 se pegarmos dez livros de cálculo isso não ocorre, concordam!?

Talvez a observação seja livros de cálculo são mais formatados do que
livros de álgebra linear. E, do meu ponto de vista, isso é ruim pro
cálculo. É claro que a homogeneidade simplifica muito a transferência
(tanto de professores quanto de alunos), e talvez até simplifique o
ensino (afinal, se todo mundo pensa da mesma forma e aprendeu da mesma
forma, é mais fácil de tirar dúvidas), mas eu acho que empobrece a
matéria como um todo. Além disso, quem garante que o roteiro
tradicional é o melhor para todos os alunos? Não seria melhor ter
abordagens diferentes para pessoas que pensam diferente?

-- 
Bernardo Freitas Paulo da Costa

-- 
Esta mensagem foi verificada pelo sistema de antivírus e
 acredita-se estar livre de perigo.


=
Instruções para entrar na lista, sair da lista e usar a lista em
http://www.mat.puc-rio.br/~obmlistas/obm-l.html
=


Re: [obm-l] Integral

2013-07-28 Por tôpico Bernardo Freitas Paulo da Costa
2013/7/28 Artur Costa Steiner steinerar...@gmail.com:
 Numa primeira análise, podemos afirmar que esta integral existe. Para x  1, 
 |(e^(-x) - e^(-ex))/x|  1. Como a integral de 1 a oo de e^(-x) - e^(-ex) 
 claramente existe e é finita, a sua integral existe e é finita em [1, oo). Na 
 realidade, é positiva, pois o integrando é positivo.

 Assim, se a integral imprópria sobre (0, 1] for finita, a integral sobre (0, 
 oo) existirá. Temos que o integrando é uma função contínua em (0, 1] e que  
 lim x -- 0+ (e^(-x) - e^-ex)/x =  e - 1. Logo, a função é limitada em (0, 
 1]. Se vc extender o domínio para [0, oo) definindo g(0) = e - 1, vc obtém 
 uma função contínua, logo integrável, em [0, 1]. E esta integral é igual à 
 integral imprópria da função original sobre (0, 1].

 Assim, sua integral existe e é finita sobre (0, oo). Mas determiná-la,  não 
 parece uma tarefa fácil. Achar a primitiva em forma fechada, acho que não dá.

Queremos integrar [ exp(-x) - exp(-ex) ]/x de 0 a infinito, e o Artur
já fez o favor de mostrar que a integral existe. Agora é só calcular.
Vamos por integrais impróprias, mesmo que eu ache que deve ter uma
solução usando resíduos:

I = limite eps-0 int_eps^infinito  [ exp(-x) - exp(-ex) ]/x dx. Chame
essa integral de I_eps.

I_eps = int_eps^infinito exp(-x)/x dx - int_eps^infinito exp(-ex)/x dx
= I_eps_1 - I_eps_2.

Agora, faça uma mudança de variáveis y = ex na segunda integral, ela vira

I_eps_2 = int_eps^infinito exp(-ex)/x dx = int_(e * eps)^infinito
exp(-y)  dy (Note que dx/x = dy/y para y = k*x, qualquer que seja k).

Assim, I_eps = integral de eps até (e * eps) exp(-x) dx/x = integral
de 1 até e de exp(-u*eps) du/u (para x = u*eps, mesma observação
anterior)

Mas quando eps-0, o integrando tende (uniformemente ! em [1,e]) à
função 1/u. E essa integral é o logaritmo de e, ou seja, 1.

Abraços,

-- 
Bernardo Freitas Paulo da Costa

-- 
Esta mensagem foi verificada pelo sistema de antivírus e
 acredita-se estar livre de perigo.


=
Instruções para entrar na lista, sair da lista e usar a lista em
http://www.mat.puc-rio.br/~obmlistas/obm-l.html
=


[obm-l] Re: [obm-l] Re: [obm-l] Divisibilidade(congruência)

2013-07-10 Por tôpico Bernardo Freitas Paulo da Costa
2013/7/11 Eduardo Wilner eduardowil...@yahoo.com.br

 A formulação não está correta; contra-exemplo : m=3 e n= 9

3*9 = 27, mais um, 28. Não vejo problema.

 De: marcone augusto araújo borges marconeborge...@hotmail.com
 Para: obm-l@mat.puc-rio.br obm-l@mat.puc-rio.br
 Enviadas: Quarta-feira, 10 de Julho de 2013 22:17
 Assunto: [obm-l] Divisibilidade(congruência)

 Sejam m e n dois números naturais tais que mn + 1 é multiplo de 24.
 Mostre que m + n tambem é múltiplo de 24.

 Se possivel,gostaria que alguem resolvesse por congruencia.Obrigado.

Vou mostrar a parte divisível por 3, você faz a por 8: como 3
divide mn + 1, temos que nem m nem n são divisíveis por 3, logo valem
1 ou 2 módulo 3. Mas note que se m*n = -1 mod 3, então não pode
ocorrer m=n mod 3 (porque então m*n seria 1 mod 3). Assim, m = 1, n =
2, ou o contrário. Logo, m+n = 1+2 = 0 mod 3.

--
Bernardo Freitas Paulo da Costa

-- 
Esta mensagem foi verificada pelo sistema de antivírus e
 acredita-se estar livre de perigo.


=
Instruções para entrar na lista, sair da lista e usar a lista em
http://www.mat.puc-rio.br/~obmlistas/obm-l.html
=


[obm-l] Re: [obm-l] Equações funcionais

2013-06-29 Por tôpico Bernardo Freitas Paulo da Costa
2013/6/29 João Maldonado joao_maldona...@hotmail.com:
 Meu professor me passou uma lista de equações funcionais e teve 3 problemas
 que eu não consegui fazer, ficaria grato se vocês me dessem uma mão

 3) (IMO) Seja Q+ o conjunto dos reais positivos. Construa uma função f:Q+ -
 Q+ tal que f(x f(y)) = f(x)/y, qualquer que seja x e y pertencentes Q+
 DICAS:
 a) Prove que a função é injetora
 b) Prove que a função é multiplicativa

 No terceiro eu consegui provar que a função é injetora e bijetora, e com
 isso consegui provar que também é multiplicativa, ou seja, f(a.b) =
 f(a).f(b)
 Mas daí eu apliquei a quarta equação funcional de Cauchy, que tem como
 solução f(x) = x^k, achando k = i, absurdo

Na verdade, você achou que f(f(f(f(x = x, né? (f composta 4 vezes)
E também f(f(x)) nem sempre é igual a x, então não dá uma função tão
simples como f(x) = 1/x, que é a tentativa inicial (pelo menos pra
mim).

 Eu andei pesquisando na internet e vi que existem outras soluções para a
 equação de Cauchy além da trivial, e x^k obviamente não dá certo. Como
 poderia construir uma função que satisfaça o exercício?

Como f é multiplicativa, basta ver como ela funciona nos números
primos, e como ela funciona para inversos (multiplicativos) f(1/x) = ?
Daí, tente achar um jeito de driblar a raiz quarta da unidade.

-- 
Bernardo Freitas Paulo da Costa

-- 
Esta mensagem foi verificada pelo sistema de antivírus e
 acredita-se estar livre de perigo.


=
Instruções para entrar na lista, sair da lista e usar a lista em
http://www.mat.puc-rio.br/~obmlistas/obm-l.html
=


[obm-l] Re: [obm-l] Re: [obm-l] Álgebra Linear

2013-06-26 Por tôpico Bernardo Freitas Paulo da Costa
2013/6/26 terence thirteen peterdirich...@gmail.com:
 Se não estou enganado, é só fazer a mesma transformação na matriz
 identidade. A matriz resultante seria aquela que faz a transformação que
 você quer. É um truque um tanto sujo, mas acho que dá para demonstrar
 isto...

Depende. Você trocar sub-linhas me parece mais difícil.

Por exemplo,

A = [1, 2, 3 ; 4, 5, 6; 7, 8, 9]

Eu quero trocar o 2 com o 8. Fazendo isso na identidade, você trocou
dois zeros, e não é bem isso. E se você quiser trocar o [2,3] com o
[8, 9], e transformar a identidade, você acaba na verdade trocando
[1,2,3] com [7,8,9]...

Fazendo a transformação agir na identidade, você obtém uma matriz M.
Multiplicando por M de um lado, você troca linhas, do outro, colunas.
Mas sempre inteiras, não sub-coisas. Acho que deve dar pra provar
que transformações lineares que trocam sub-linhas/colunas não existem.
Quer dizer, sem nem pedir que seja ortogonal.

Abraços,
-- 
Bernardo Freitas Paulo da Costa

 Em 26 de junho de 2013 14:35, Kurstchak kurstc...@globo.com escreveu:

 Amigos,

 é possivel fazer uma transformacao ortogonal que troque (sub) linhas
 (colunas) de uma sub matriz, preservando os demais elementos?

 Agradeço antecipadamente !

 CArlos


-- 
Esta mensagem foi verificada pelo sistema de antivírus e
 acredita-se estar livre de perigo.


=
Instruções para entrar na lista, sair da lista e usar a lista em
http://www.mat.puc-rio.br/~obmlistas/obm-l.html
=


[obm-l] Re: [obm-l] equação diferencial dúvida

2013-06-19 Por tôpico Bernardo Freitas Paulo da Costa
2013/6/19 Hermann ilhadepaqu...@bol.com.br:
 Considere a eq dif

 y' = (2x + x.cos(x))/2y

 y = x + x.cos(x)/2 é uma solução para esta eq dif?

 Cheguei na seguinte equação cos(x)-x.sen(x)=2 e travei.
Normal, porque a função (x + x*cos(x)/2) não é solução.

Para uma eq dif dada por F(x,y,y') = 0, y(x) será uma solução se, ao
substituir o y pela expressão em x, você obtiver uma identidade (ou
seja, válida para todo x).

Abraços,
-- 
Bernardo Freitas Paulo da Costa

-- 
Esta mensagem foi verificada pelo sistema de antivírus e
 acredita-se estar livre de perigo.


=
Instruções para entrar na lista, sair da lista e usar a lista em
http://www.mat.puc-rio.br/~obmlistas/obm-l.html
=


[obm-l] Re: [obm-l] Como mostrar que esta integral é um imaginário puro

2013-06-17 Por tôpico Bernardo Freitas Paulo da Costa
2013/6/17 Merryl M sc...@hotmail.com:
 Oi amigos,
Oi,

 Seja f definida no plano complexo por f(z) = f(x + yi) = u(x) + i v(y),
 sendo u e v funções contínuas de R em R. Mostre que, para toda curva suave e
 fechada c do plano, Int_c f(z) dz é um imaginário puro.

Basta escrever dz = dx + i dy, expandir e mostrar que a parte real tem
integral zero porque a curva é fechada.

 Se u e v forem diferenciáveis, então esta integral tem que ser nula?
Não, basta escolher u e v tais que du/dx != dv/dy. Por exemplo, u(x) =
2x, v(y) = 0. Note, claro, que f assim feita não será holomorfa.

-- 
Bernardo Freitas Paulo da Costa

-- 
Esta mensagem foi verificada pelo sistema de antivírus e
 acredita-se estar livre de perigo.


=
Instruções para entrar na lista, sair da lista e usar a lista em
http://www.mat.puc-rio.br/~obmlistas/obm-l.html
=


Re: [obm-l] potencia de base irracional

2013-06-16 Por tôpico Bernardo Freitas Paulo da Costa
2013/6/16 marcone augusto araújo borges marconeborge...@hotmail.com:
 não tenho ideia de como se resolve essa

 mostre que os 1000 dígitos após a virgula decimal de(8 +65^1\2)^1\2 são
 todos iguais a 9

Isso é raiz( 8 + raiz(65) ) ? Se for, isso dá
4.0077746628644866530696276028515203551928... e portanto não tem nada
a ver com o enunciado...
-- 
Bernardo Freitas Paulo da Costa

-- 
Esta mensagem foi verificada pelo sistema de antivírus e
 acredita-se estar livre de perigo.


=
Instruções para entrar na lista, sair da lista e usar a lista em
http://www.mat.puc-rio.br/~obmlistas/obm-l.html
=


[obm-l] Re: [obm-l] Função crescente e derivável tal que f tem limite real mas f' não vai para 0

2013-05-31 Por tôpico Bernardo Freitas Paulo da Costa
2013/5/31 Artur Costa Steiner steinerar...@gmail.com:
 Olá amigos!

 Isto não parece muito difícil, mas até agora não consegui.

 Exemplo de uma função de R em R (ou definida em (a, oo) para algum a) que 
 seja crescente e derivável, seja tal que lim x -- oo f(x) = L em R e tal que 
 a condição lim x -- oo f'(x) = 0 não se verifique.

 Como f' tendo limite positivo no infinito implicaria que f fosse para 
 infinito, então f' não pode ter limite no infinito, tem que ficar oscilando.

 Se vc não exigir que f seja monotônica, não é difícil achar um exemplo. f(x) 
 = (sen(x^2))/x , x  0, atende. f vai para 0 no infinito e f'(x) = 2 cos(x^2) 
 - (sen(x^2))/x fica oscilando e não converge para nada. Mas f' assume uma 
 infinidade de valores positivos e uma infinidade de negativos, de modo que f 
 não é monotônica.

 No nosso caso, temos que garantir que f' fique oscilando mas sem assumir 
 valores negativos. Acho que f tem que ser um tanto patológica.

Não sei o que você chama de patológica, mas você já tem a solução,
falta só descrever a solução sem ser com uma fórmula analítica
bonitinha.

Sejam então a_n = 2^n, b_n = a_n + 4^(-n), c_n = 5^(-n).

Considere a função que vale 0 em a_0, é linear com derivada 1 até b_0,
e depois linear com derivada c_0 até a_1. Em geral, ela é linear com
derivada 1 entre a_n e b_n, e linear com derivada c_n entre b_n e
a_{n+1}.

Pronto, temos uma função crescente, derivável (quase), a derivada não
tende a zero (como você mesmo disse) mas com limite: de a_n até a_n+1
a função cresce menos de 4^(-n) + (2/5)^n, que é dá uma série
convergente. Agora, basta aparar os cantos da função para obter uma
função C-infinito.

Talvez isso dê uma idéia de como pode-se construir uma fórmula
bonitinha para f, mas isso eu deixo pra você.

Abraços,
-- 
Bernardo Freitas Paulo da Costa

=
Instruções para entrar na lista, sair da lista e usar a lista em
http://www.mat.puc-rio.br/~obmlistas/obm-l.html
=


Re: [obm-l] Soma igual ao produto

2013-05-11 Por tôpico Bernardo Freitas Paulo da Costa
2013/5/11 Ralph Teixeira ralp...@gmail.com:
 Bom, se voce deixar a pergunta assim, a resposta eh sim, montes deltes.

 Afinal, 1+1+1+...+1+x_1+x_2+...+x_n=1.1.1.1.1.1.1.x_1.x_2.x_3x_n se
 voce botar o numero certo de 1's ali...

 Entao a pergunta bacana eh...?

Poxa, eu achei 1 + 2 + 3 = 1 * 2 * 3 tão bacana!
-- 
Bernardo Freitas Paulo da Costa

=
Instruções para entrar na lista, sair da lista e usar a lista em
http://www.mat.puc-rio.br/~obmlistas/obm-l.html
=


Re: [obm-l] teoria dos numeros

2013-05-10 Por tôpico Bernardo Freitas Paulo da Costa
2013/5/10 terence thirteen peterdirich...@gmail.com:
 Aprenda um pouco de inglês:

 http://ohkawa.cc.it-hiroshima.ac.jp/www.kalva.demon.co.uk/apmo/asoln/asol972.html


 Em 10 de maio de 2013 06:48, valdir soares valdir.soa...@oi.com.br
 escreveu:

 Ola pessoal,

 Gostaria de saber, como fazer o problema abaixo :

 Determine n entre 100 e 1000 , tal que   ( 2+ 2^n)/n  eh tambem inteiro .

Braço por braço (a solução contando os primos e verificando que 2 é
primitiva módulo p, e depois mais braço para p, 2p, pq, 2pq), dá pra
rodar todos esses números. E (ao contrário do kalva) eu usei a
calculadora do linux em linha de comando:

$ bc
define r(n) { n ; return (2 + 2^n) % n }
for (i = 100; i = 1000; i++) r(i)

Depois, com a resposta na mão, você apenas verifica que dá certo ;-)
-- 
Bernardo Freitas Paulo da Costa

=
Instruções para entrar na lista, sair da lista e usar a lista em
http://www.mat.puc-rio.br/~obmlistas/obm-l.html
=


[obm-l] Re: [obm-l] Polígono regular inscrito

2013-05-01 Por tôpico Bernardo Freitas Paulo da Costa
2013/5/1 Martins Rama martin...@pop.com.br:
 Caros amigos da lista...
 A afirmação abaixo é verdadeira? Como prová-la? Indução, talvez?

 Para um polígono regular convexo de n vértices V1, V2, ...,Vn, inscrito
 num círculo de raio unitário, qual o valor do produto das medidas das
 (n-1) cordas traçadas de um vértice, por exemplo, V1?

 P = V1V2 x V1V3 x ... x V1Vn = ?

 Sei que para o:
 - triângulo equilátero, temos: (raiz de 3)x(raiz de 3) = 3
 - quadrado, temos: (raiz de 2)x(raiz de 2)x2 = 4
 - hexágono regular, temos: 1x(raiz de 3)x2x(raiz de 3)x1 = 6

 É possível generalizar a solução e encontrar a resposta n para todos os
 polígonos regulares?

Isso é verdade. Eu não conheço nenhuma demonstração por indução, mesmo
porquê veja que os pontos do polígono regular de (n+1) lados não tem
nada a ver com os de n lados (ou praticamente nada a ver). Eu sei uma
com raízes da unidade, e que faz aparecer o produto das cordas numa
relação de Girard.

Abraços,
-- 
Bernardo Freitas Paulo da Costa

=
Instruções para entrar na lista, sair da lista e usar a lista em
http://www.mat.puc-rio.br/~obmlistas/obm-l.html
=


[obm-l] Re: [obm-l] Sistema de Três Equações com Quadrados

2013-05-01 Por tôpico Bernardo Freitas Paulo da Costa
2013/5/1 terence thirteen peterdirich...@gmail.com:

 Resolva o sistema abaixo:

 3(S-l)^2+D^2=3^2
 3S^2+(l-D)^2=4^2
 3S^2+(l+D)^2=5^2
Dá uns números muito feios?

III - II elimina tudo menos 4 l D = 25 - 16 = 9.

Daí, II - I elimina quase tudo menos 6 S l - 2 D l = 7, mas a gente
tem 4 D l do anterior. Substitui D = 9/4l e S = 23/12l, e obtenha uma
biquadrada... e coragem com as raízes.

-- 
Bernardo Freitas Paulo da Costa

=
Instruções para entrar na lista, sair da lista e usar a lista em
http://www.mat.puc-rio.br/~obmlistas/obm-l.html
=


Re: [obm-l] Problema 4 Cone Sul 1996

2013-04-30 Por tôpico Bernardo Freitas Paulo da Costa
  Se escolhem
  dois ou mais números da seqüência (mas não todos) e se sustitui um deles 
  pela
  média aritmética dos números escolhidos, obtendo-se assim uma nova 
  seqüência de
  1996 números.
 
2013/4/30 Carlos Yuzo Shine cysh...@yahoo.com:
 já que a soma de todos nunca muda

Confesso que não entendi direito. Imagine que você escolhe os 4
primeiros números, 0, 1, 1, 1. Qual é o resultado da operação? Da
forma como o enunciado parece indicar, isso DEPENDE de uma OUTRA
escolha, a saber a do um deles a ser substituído por 3/4. Ou seja,
podemos ficar com 3/4, 1, 1, 1; ou 0, 3/4, 1, 1 (ou uma permutação). O
Shine parece argumentar que o resultado será 3/4, 3/4, 3/4, 3/4. Mas
isso só seria o caso se estivesse escrito .. se substitui CADA UM
deles ..., não?

-- 
Bernardo Freitas Paulo da Costa

=
Instruções para entrar na lista, sair da lista e usar a lista em
http://www.mat.puc-rio.br/~obmlistas/obm-l.html
=


Re: [obm-l] Problema de Geometria

2013-04-28 Por tôpico Bernardo Freitas Paulo da Costa
Eu vi essa parte da solução:

2013/4/28 Carlos Victor victorcar...@globo.com:
 2) Trace BD

Daí, eu vi que E = G é o baricentro de ABD. Logo OE = OD/3. Como FD =
OD - a/2, porque OF = a/2 é o raio do círculo, acabou.

Abraços,
-- 
Bernardo Freitas Paulo da Costa

=
Instruções para entrar na lista, sair da lista e usar a lista em
http://www.mat.puc-rio.br/~obmlistas/obm-l.html
=


[obm-l] Re: [obm-l] Re: [obm-l] Polinômio que mapeia racionais sobrejetivamente

2013-04-27 Por tôpico Bernardo Freitas Paulo da Costa
2013/4/27 Marcelo Salhab Brogliato msbro...@gmail.com:
 Lucas, boa tarde!

 Se entendi corretamente sua questão, p é linear. Seja I = [a, b] e J = [c,
 d], então, p é a reta que passa pelos pontos (a, c) e (b, d). Ou seja, p(x)
 = c + [ (d - c) / (b - a) ] * (x - a). Veja que p(a) = c e p(b) = d.
E porquê não a reta (a,d) -- (b,c) ?

Se eu me lembro bem, esse problema é relativamente sutil.
-- 
Bernardo Freitas Paulo da Costa

=
Instruções para entrar na lista, sair da lista e usar a lista em
http://www.mat.puc-rio.br/~obmlistas/obm-l.html
=


Re: [obm-l] Matrizes

2013-04-26 Por tôpico Bernardo Freitas Paulo da Costa
2013/4/26 Athos Cotta Couto cotta.co...@gmail.com:
 É meio pesado isso aí ein!?
 A diferença é que o problema que o Tao trata usa -1 e 1 como entradas,
 eu to analizando 0 e 1. Tomara que essa mudança cause uma diferença grande
 de dificuldade...
Acho que não muda muito o problema, aposto que se forem dois números
quaisquer x e y em vez de 0 e 1 ou -1 e 1 deve ser equivalente, para n
grande. Se você tiver tempo, use um programa para calcular todos os
casos possíveis. São 2^(n^2), pra ser razoável digamos que isso dê
menos de 10^9, ou seja mais ou menos n^2 = 30, o que dá n=5. Veja
como fica a probabilidade para n=1, n=2, n=3, n=4, n=5 e mande ver na
OEIS ...

-- 
Bernardo Freitas Paulo da Costa

=
Instruções para entrar na lista, sair da lista e usar a lista em
http://www.mat.puc-rio.br/~obmlistas/obm-l.html
=


[obm-l] Re: [obm-l] Inversão de Matrizes

2013-04-26 Por tôpico Bernardo Freitas Paulo da Costa
2013/4/26 Athos Cotta Couto cotta.co...@gmail.com:
 Mostre que se I-AB é invertível, então I-BA também é.

Escreva a série de 1/(1 - x) =  1 + x + x^2 + x^3 + ...

Substitua x = AB (eq. 1) e depois x = BA (eq. 2).

Chame 1/(1 - AB) de M. Chame (a série) de 1/(1 - BA) de N. Tente obter
N multiplicando e somando a partir de M, sem dividir.

Daí, isso vai dar uma fórmula para N, em função de M, a inversa (que
existe) de 1 - AB. Eu obtive N = BMA + 1. Agora, prove que essa é
realmente a inversa, multiplicando por 1 - BA, e usando que M(1 - AB)
= 1, ou seja M - MAB = 1, ou seja, MAB = M - 1. Você provavelmente
também vai precisar de ABM = M - 1, para mostrar que é a inversa do
outro lado.

Abraços,
-- 
Bernardo Freitas Paulo da Costa

=
Instruções para entrar na lista, sair da lista e usar a lista em
http://www.mat.puc-rio.br/~obmlistas/obm-l.html
=


[obm-l] Re: [obm-l] Definição de número

2013-04-22 Por tôpico Bernardo Freitas Paulo da Costa
2013/4/22 Fernando Villar villarferna...@gmail.com:
 Boa tarde, amigos.

 Há uma boa definição para números que englobe desde os naturais até os
 complexos?
Tudo que você pode somar, subtrair, multiplicar e dividir e contém
1?  (O 1 taí só pra evitar um cara chato que invente de vir com
matrizes quadradas - mas matrizes quadradas são quase como números).

Por outro lado, porque un número natural não seria um número
complexo e portant o número seria apenas uma abreviação de número
complexo para nós preguiçosos?
-- 
Bernardo Freitas Paulo da Costa

-- 
Esta mensagem foi verificada pelo sistema de antivírus e
 acredita-se estar livre de perigo.


=
Instruções para entrar na lista, sair da lista e usar a lista em
http://www.mat.puc-rio.br/~obmlistas/obm-l.html
=


Re: [obm-l] Um belo determinante!!!(ajuda)

2013-04-22 Por tôpico Bernardo Freitas Paulo da Costa
2013/4/22  douglas.olive...@grupoolimpo.com.br:
 Olá meus amigos , caso tenham algum tempinho pra me ajudar a pensar neste
 incrível determinante,

 O resultado é impressionante!!
Realmente. Eu ainda não provei, mas eu já sei que é o produto de
phi(k) k = 1 até n.

 QUESTÃO:

 calcular o determinante de uma matriz nxn , onde cada elemento é exatamente
 o MDC entre i e j. sendo i o número da linha e j o número da coluna.

Sugestão: elimine de baixo pra cima, que nem Vandermonde (posso
parecer repetitivo, mas o é que funciona...)
-- 
Bernardo Freitas Paulo da Costa

-- 
Esta mensagem foi verificada pelo sistema de antivírus e
 acredita-se estar livre de perigo.


=
Instruções para entrar na lista, sair da lista e usar a lista em
http://www.mat.puc-rio.br/~obmlistas/obm-l.html
=


[obm-l] Re: [obm-l] Re: [obm-l] Definição de número

2013-04-22 Por tôpico Bernardo Freitas Paulo da Costa
 A busca é por uma definição que sirva para contextos de alfabetização até 
 contextos do 3o ano do EM, quando se ensina, usualmente, números complexos.
Ah, você quer uma definição matemática, axiomática e tal? Para
construir os números (a partir de outros conceitos primitivos, sejam
lá quais forem)? Eu acho que isso é meio impossível, as definições
construtivas são sempre por etapas.

Para mim, a idéia das operações básicas é o fio condutor. Então,
para os pequenos e para os grandes, o que é importante é ver que você
soma / subtrai / multiplica / divide, e o que cada uma dessas
operações traduz. Confesso que os complexos são uma baita abstração (e
muito tempo evitados como imaginários e outros termos pejorativos)
mas talvez, justamente, isso tenha a ver com uma operação muito mais
difícil do que as anteriores: resolver equações de segundo grau! Note
que os reais também vêm de uma operação complicada: limites! Mas é
uma idéia muito muito mais fácil de intuir do que x a solução de x^2
+ x + 5 = 0 (Não digo de formalizar, os complexos nesse sentido foram
formalizados antes dos números reais!). Claro que tem o plano
complexo, coordenadas simples, mas ainda assim é meio estranho. O
que devolve a naturalidade dos números complexos é a série de Taylor,
que junta polinômios e limites, mas daí a gente já tá entrando na
Universidade. Eu acho fundamental fazer os alunos notarem que tudo
faz sentido quando você junta as coisas em matemática, e não quando
você as mantém isoladas!

 O que acontece é que, na matemática, geralmente nos procupamos muito mais
 com as relações entre objetos matemáticos do que  com suas naturezas.
Acho importante não perder muito de vista a natureza dos objetos, mas
principalmente a sua natureza intuitiva, mais do que a pura definição
formal que assenta a solidez do edifício matemático. Claro que é
importante ter uma definição formal e tudo, para justamente
fundamentar o edifício, mas não é a definição formal que vai te
ajudar a descobrir algo novo, ou mesmo entender algo antigo.

-- 
Bernardo Freitas Paulo da Costa

-- 
Esta mensagem foi verificada pelo sistema de antivírus e
 acredita-se estar livre de perigo.


=
Instruções para entrar na lista, sair da lista e usar a lista em
http://www.mat.puc-rio.br/~obmlistas/obm-l.html
=


[obm-l] Re: [obm-l] Re: [obm-l] Re: [obm-l] Definição de número

2013-04-22 Por tôpico Bernardo Freitas Paulo da Costa
2013/4/22 Fernando Villar villarferna...@gmail.com:
 Isso surgiu de uma discussão sobre a expressão números irracionais são
 aqueles que não podem ser escritos como a razão de números inteiros. Houve
 um questionamento de que tal definição incluiria os complexos como
 irracionais.
 Daí surgiu a dúvida se na expressão números complexos haveria ou não uma
 apropriação indevida do termo números.
Não! Enfim, para mim, absolutamente, não. O problema é a sua definição
de números irracionais. Com a sua definição, sim, i é irracional.
Mas eu sugiro uma outra: um número irracional é um número real que não
é razão de dois números inteiros.

Sendo beem honesto: definições negativas são um porre ;-). O
importante é o conceito de racional. O irracional é o resto, e o
resto é sempre mal-definido. Um quatérnion é um número? E uma oitava
(de Cayley)? O mais importante (para os seus alunos) é eles entenderem
que a questão 1 + 3i é irracional é simplesmente fora de
propósito. É uma simples questão de gosto, e nada (insisto, nada) na
matemática será avançado com uma resposta a essa questão, porque ela
não dá mais entendimento sobre os números complexos, ou reais, ou
racionais, ou sobre o 1 + 3i em particular. Por outro lado, os seus
alunos terem a maturidade de considerar a questão, estudar o sentido
da mesma, e em seguida descartá-la como inapropriada, isso sim, os
terá feito entender o sentido da matemática: algumas definições são
cruciais, outras mera convenção (com 0 é natural) e outras resíduos
históricos. Compreendê-las como tais, e reconhecer que por conta disso
algumas questões não são epistemologicamente apropriadas, é um
GRANDE passo.

Abraços,
-- 
Bernardo Freitas Paulo da Costa

-- 
Esta mensagem foi verificada pelo sistema de antivírus e
 acredita-se estar livre de perigo.


=
Instruções para entrar na lista, sair da lista e usar a lista em
http://www.mat.puc-rio.br/~obmlistas/obm-l.html
=


Re: [obm-l] Coisas simples mas complicadas para explicar

2013-04-22 Por tôpico Bernardo Freitas Paulo da Costa
2013/4/22 Artur Costa Steiner steinerar...@gmail.com:
 Isso é uma curiosidade, mas eu me lembrei porque estão um tanto na linha 
 daquela discussão do que é número. São duas situações reais.

 1) Qual é a soma das raízes da equação (x - 1)^4 = 0. Alguns dizem 4, 
 alegando haver 4 raízes iguais a 1. E as relações de Girard, que considera 
 multiplicidades, leva a 4. Mas o fato é que o conjunto verdade desta equação 
 é {1}. 1 é o único complexo que a satisfaz. Então outros dizem que a soma é 
 1. Mas, soma é uma operação binária, não existe soma de uma só parcela. O que 
 acontece é que quando se usa o sigma maiúsculo do somatório, convenciona-se 
 que se na lista só houver um elemento, então o resultado é o próprio. Mas 
 soma de uma parcela não faz sentido. Como vc responderia a esta pergunta?

As relações de Girard são algébricas, portanto elas naturalmente
incorporam multiplicidades. Além disso, elas são contínuas, o que é
muito bom. Nada disso ocorre com uma definição a soma das raízes sem
contar multiplicidades. Então, da mesma forma que a gente começa com
um ensaio de definição para o número de raízes de um polinômio (o
cardinal do tal conjunto), o objetivo é refinar a definição para dizer
que um polinômio de grau d tem d raízes (e o contadas com
multiplicidade passa a ser a forma natural de se pensar nas raízes do
polinômio). Isso tem uma razão profunda: a alternativa, ou seja, não
contar multiplicidades, possui muito menos informação, e portanto
teoremas. Existe uma parte (relativamente arcana) de análise complexa
que tenta contar raízes e outras coisas sem multiplicidade, mas ela
tem de longe muito menos generalidade e portanto aplicações do que as
relações de Girard. E isso sem falar de funções reais (não
analíticas), onde os teoremas de índice (Bott, Homologia ...)
incorporam noções ainda mais complicadas de multiplicidade
(negativas!) por questões de orientação...

Quanto à notação de somatório, isso é fácil: isso é convenção para
uniformizar a notação. \sum_J x_j = 0 para J o conjunto vazio,
definida por indução no número de elementos de J, e definida como um
limite (ordenado ou não, depende de como você quer escrever os
índices, e tem a ver - claro - com absolutamente ou condicionalmente
convergente) no caso de J ser infinito. Aqui, a resposta tem mais a
ver com levar o absurdo mais adiante ainda !

 2) Uma moça disse: não tenho filhos. Logo, por vacuidade, posso afirmar que 
 meu filho é um profundo conhecedor de análise complexa. Se eu afirmar isto, 
 não estarei mentindo. Mas se alguém disser, puxa que bom! Tenho umas dúvidas 
 nisso, ele pode me ajudar?, vai ficar frustrado e chateado comigo.

 Por outro lado, como meus filhos não existem, alguém pode dizer que eu os 
 matei. Sou infanticida por vacuidade. Apesar de que nenhuma corte me 
 condenaria  por matar quem não existe.

 O que acham disso?
Eu acho que é um problema de quantificadores e de abuso de notação. A
moça não pode dizer meu filho é um profundo conhecedor de análise
complexa, porque isso é equivalente à proposição P(meu filho). O
que ela pode dizer é todos os meus filhos são profundos conhecedores
de análise complexa, porque, agora sim, você tem uma proposição
quantificada que pode, portanto, ser verdadeira por vacuidade: para
todo x pertencente a F, P(x). Note que, dada esta afirmação, a
pergunta seguinte seria, bem naturalmente, Será que tem um deles que
pode me ajudar?. E veja que (claro!) a resposta é não, porque toda
proposição existencial sobre o conjunto vazio é falsa.

A segunda é essencialmente a mesma. Você matou todos os seus filhos
não é a definição de um infanticida, mas esta pessoa matou crianças,
que exige a construção (ou demonstração de existência, pelo menos) de
uma criança que tenha sido morta. Ainda bem!

Ou seja, antes de sair usando lógica demais, lembre que o seu
bom-senso deve ajudar na hora de decidir se algo é verdade ou não.
Claro que para todo x em vazio, P(x) é o mais difícil de intuir,
mas, honestamente, quase nunca (na mesma linha da discussão anterior)
faz muito sentido ficar demonstrando algo sobre o conjunto vazio,
porque, no bom e velho mundo real, não adianta! (Às vezes, acontece de
alguém fazer um seminário sobre o conjunto vazio... desta forma, não
menosprezem a pergunta você tem um exemplo de aplicação disso,
porque pode ser que você esteja com tantas hipóteses que, no fim das
contas, você demonstrou um teorema sobre o conjunto vazio!)

Abraços,
-- 
Bernardo Freitas Paulo da Costa

-- 
Esta mensagem foi verificada pelo sistema de antivírus e
 acredita-se estar livre de perigo.


=
Instruções para entrar na lista, sair da lista e usar a lista em
http://www.mat.puc-rio.br/~obmlistas/obm-l.html
=


Re: [obm-l] Torneio das Cidades 94

2013-04-20 Por tôpico Bernardo Freitas Paulo da Costa
2013/4/20 terence thirteen peterdirich...@gmail.com:
 Então, façam o seguinte: alguém tem o enunciado originbal em russo ou em
 inglês? Daria para saber daí se ele fala avós como plural de avó ou como
 plural de avôs.

 Acho que se for dois, o problema fica mais fácil.
Como eu escrevi, se forem 4, o problema tem *outra* solução.

 Mas só comentar:

 'Portanto, existe uma avo' Odete, por exemplo, comum aos alunos dos grupos B
 e C.'

 Na verdade B e C são grupos, e cada par B-C pode ter uma avó distinta. Isso
 dificulta...
Não: cada aluno de B tem que ter uma avó comum com cada aluno de C.
Como não é a primeira avó, tem que ser a outra. Com os pingos nos
is: seja Odete a avó de b1 e c1. Como b1 tem uma avó comum com todos
os alunos de C, e b1 - (Maria, Odete) e ci - (Nair, ???), concluímos
que ??? = Odete para todos os alunos de C. Da mesma forma, c1 -
(Nair, Odete) e bi - (Maria, ), logo  = Odete também.

Abraços,
-- 
Bernardo Freitas Paulo da Costa

-- 
Esta mensagem foi verificada pelo sistema de antivírus e
 acredita-se estar livre de perigo.


=
Instruções para entrar na lista, sair da lista e usar a lista em
http://www.mat.puc-rio.br/~obmlistas/obm-l.html
=


Re: [obm-l] determinantes 02

2013-04-19 Por tôpico Bernardo Freitas Paulo da Costa
2013/4/19 Thelio Gama teliog...@gmail.com:
 Tem mais essa outra anexa parecida com a anterior que postei, e que também
 não consigo resolver.
Esse é um Vandermonde normal. Escreva tudo, sai fatorado, e depois
corra pro abraço com a análise de sinal. É chato e longo, mas não é
difícil.
-- 
Bernardo Freitas Paulo da Costa

-- 
Esta mensagem foi verificada pelo sistema de antivírus e
 acredita-se estar livre de perigo.


=
Instruções para entrar na lista, sair da lista e usar a lista em
http://www.mat.puc-rio.br/~obmlistas/obm-l.html
=


Re: [obm-l] determinantes 01

2013-04-19 Por tôpico Bernardo Freitas Paulo da Costa
2013/4/19 Thelio Gama teliog...@gmail.com:
 Prezados professores,

 tentei mas não consegui resolver  a equação anexa. Parecia-me uma matriz de
 Wandermonde, mas não consegui. Poderiam me ajudar?

Eu aposto que é um erro tipográfico. Mas, se não for, use eliminação
de baixo pra cima, como se fosse Vandermonde (ou seja, subtraia a
linha 3 na linha 4, depois a linha 2 na linha 3, depois a linha 1 na
linha 2), obtendo assim três zeros na última coluna, e daí basta
calcular o determinante da matriz que sobrar. Vai dar um polinômio, um
monte de fatorações igual ao que acontece no Vandermonde vai aparecer,
mas no fim resta um polinômio (k^3 - k - 1) feioso...

-- 
Bernardo Freitas Paulo da Costa

-- 
Esta mensagem foi verificada pelo sistema de antivírus e
 acredita-se estar livre de perigo.


=
Instruções para entrar na lista, sair da lista e usar a lista em
http://www.mat.puc-rio.br/~obmlistas/obm-l.html
=


[obm-l] Re: [obm-l] Divisibilidade(questão simples)

2013-04-18 Por tôpico Bernardo Freitas Paulo da Costa
2013/4/18 marcone augusto araújo borges marconeborge...@hotmail.com
 Mostrar que  m = n^5 - n é divisível por 30

 Alguém resolveria por indução?
Manda um binômio de Newton em (n+1)^5, e pela hipótese de indução,
resta mostrar que

C(5,1) n + C(5,2)n^2 + C(5,3)n^3 + C(5,4)n^4 é divisível por 30.
Explicitando isso daí, você obtém:

5(n + 2n^2 + 2n^3 + n^4), que é divisível por 5 (claro!) e por 2
(número par de termos de mesma paridade que n). Pra ver módulo 3,
Fermat nele, n^3 == n, logo o treco vira

5(n + 2n^2 + 2n + n^2) = 5(3n + 3n^3), e fim.
--
Bernardo Freitas Paulo da Costa

-- 
Esta mensagem foi verificada pelo sistema de antivírus e
 acredita-se estar livre de perigo.


=
Instruções para entrar na lista, sair da lista e usar a lista em
http://www.mat.puc-rio.br/~obmlistas/obm-l.html
=


Re: [obm-l] Torneio das Cidades 94

2013-04-11 Por tôpico Bernardo Freitas Paulo da Costa
2013/4/10 Jeferson Almir jefersonram...@gmail.com:
 Existem 20 alunos em uma escola. Quaisquer dois deles possui um avó em
 comum. Prove que pelo menos 14 deles possui um avó em comum.
Só uma curiosidade... quantos avós um aluno tem? 2 ou 4?
-- 
Bernardo Freitas Paulo da Costa

-- 
Esta mensagem foi verificada pelo sistema de antivírus e
 acredita-se estar livre de perigo.


=
Instruções para entrar na lista, sair da lista e usar a lista em
http://www.mat.puc-rio.br/~obmlistas/obm-l.html
=


Re: [obm-l] Torneio das Cidades 94

2013-04-11 Por tôpico Bernardo Freitas Paulo da Costa
2013/4/11 Jeferson Almir jefersonram...@gmail.com:
 EU nao consegui resolver se puder me mandar um esboço desde ja agradeço.
Eu nem comecei a resolver porque o problema não é claro.
-- 
Bernardo Freitas Paulo da Costa

-- 
Esta mensagem foi verificada pelo sistema de antivírus e
 acredita-se estar livre de perigo.


=
Instruções para entrar na lista, sair da lista e usar a lista em
http://www.mat.puc-rio.br/~obmlistas/obm-l.html
=


Re: [obm-l] Torneio das Cidades 94

2013-04-11 Por tôpico Bernardo Freitas Paulo da Costa
2013/4/11 terence thirteen peterdirich...@gmail.com:
 Me parece bastante claro. 20 pessoas tal que, sempre que pegar duas delas,
 elas terão um avô em comum. Então existe um avô que tem 14 netos aí.
Faça pacotes de 5 alunos, A, B, C, D. Os alunos A e B têm um avô em
comum (o primeiro), C e D idem. Os alunos A e C têm um (segundo) avô
em comum, B e D idem. Finalmente, os alunos A e D têm um avô (o
terceiro) em comum, B e C idem. Assim, há ao todo 6 avôs, cada avô é
avô de 5 + 5 pessoas, cada par tem um avô em comum (trivial dentro dos
grupos, por construção fora). Portanto, talvez, sejam *apenas*
consideradas as avós, e assim cada aluno tem 2 avós, por questões de
sexo.

Abaixo problemas mal-contextualizados!
-- 
Bernardo Freitas Paulo da Costa

-- 
Esta mensagem foi verificada pelo sistema de antivírus e
 acredita-se estar livre de perigo.


=
Instruções para entrar na lista, sair da lista e usar a lista em
http://www.mat.puc-rio.br/~obmlistas/obm-l.html
=


Re: [obm-l] ITA vs. MIT

2013-04-05 Por tôpico Bernardo Freitas Paulo da Costa
2013/4/5 João Maldonado joao_maldona...@hotmail.com:
 Já faz um tempo que estou interessado em prestar ITA.
Não sei se a lista é exatamente o lugar pra perguntar isso. Mas eu
tenho uma pergunta pra você: para quê fazer ITA? Que engenharia no
ITA? Qual o seu objetivo depois de ser formado? Que tipo de engenheiro
você quer ser? Quanto você quer estudar antes de ir ao mercado de
trabalho? Pensa em fazer tese?

Eu não estudei no ITA nem no MIT, então vou responder o que eu sei.

 Quanto à admissão. 90% dos alunos do MIT são americanos. Um número bem alto
 se tratando da melhor universidade do planeta. Por que há um número tão
 baixo de estrangeiros? Será mesmo que os EUA concentram 90% das mentes mais
 brilhantes do mundo? Acho que não
Claro que não. Mas o problema é que pagar a viagem pro MIT é brabo. E
a anuidade, é pior ainda... Isso cria uma barreira muito diferente se
você é estrangeiro, e daí a maior parte dos alunos é americano
simplesmente porque é muito mais fácil entrar no MIT sendo americano.

A lenda urbana típica é que a faculdade nos EUA é uma coisa meio nas
coxas. Muita festa, pouco estudo. E como o Fundamental+Médio americano
não é nada muito genial, não dá pra fazer grandes coisas... Por outro
lado, a situação muda completamente no caso de Mestrado/Doutorado.
Primeiro, como você adivinhou, a proporção de estrangeiros dispara.
Segundo, porque a seleção é muito, muito, maior. E daí dá pra ver a
diferença, e entender porque eles são a melhor universidade de
engenharia do planeta, os laboratórios, etc.

 Quanto aos recursos. Mesmo o ITA não sendo uma universidade paga acho que
 não falta recursos (tanto para o professor quanto para o aluno0), justamente
 pela quantidade de laboratórios e o pólo tecnológico de SJC. Alguém tem
 alguma opinião?
Acho que você não tem idéia da diferença de recursos que as grandes
universidades americanas têm. É algo fenomenal. Eu visitei, apenas, o
MIT. Mas os instrumentos que eles têm por lá são coisas do outro
mundo. Talvez não faça muita diferença pruma aula de Física
Experimental (mas se for pra apostar, eu diria que há diferenças sim),
mas a graça começa se você for fazer um projeto para um curso, ou
iniciação científica, ou um projeto final.

 Quanto ao mercado de trabalho. A meu ver um diploma do MIT conta mais do que
 um do ITA (é a mesma coisa que comparar Harvard com USP). Por mais que o
 engenheiro do ITA seja laureado e tudo mais, qual dos dois formandos você
 acha que terá uma vida profissional melhor (em relação ao salário, ao tipo
 de emprego, etc.)?
Isso, eu acho, dilui após o primeiro emprego do sujeito. E, como você
pode imaginar, um bom diploma ajuda(va) a ter um bom primeiro emprego.
Daí, ambos são bons (claro, o MIT serve no mundo inteiro, não sei se
você quer viver fora do Brasil, e isso é uma questão muito, muito,
difícil).

-- 
Bernardo Freitas Paulo da Costa

-- 
Esta mensagem foi verificada pelo sistema de antivírus e
 acredita-se estar livre de perigo.


=
Instruções para entrar na lista, sair da lista e usar a lista em
http://www.mat.puc-rio.br/~obmlistas/obm-l.html
=


[obm-l] Re: [obm-l] Re: [obm-l] Espaço métrico - topologia

2013-04-04 Por tôpico Bernardo Freitas Paulo da Costa
2013/4/4 Pedro Angelo pedro.fon...@gmail.com:
 pra uma sequência desse tipo sempre vale inf |x_n - x_m|  0.
 Aí a gente usa isso pra definir f(x_n)=1/n, e estender f de
 maneira contínua pros outros pontos. Vou deixar os buracos na
 demonstração pro próximo : )

Esse buraco é mais delicado do que parece. Eu não sei fazer isso de
uma vez só / para todos os x_i ao mesmo tempo.
-- 
Bernardo Freitas Paulo da Costa

-- 
Esta mensagem foi verificada pelo sistema de antivírus e
 acredita-se estar livre de perigo.


=
Instruções para entrar na lista, sair da lista e usar a lista em
http://www.mat.puc-rio.br/~obmlistas/obm-l.html
=


[obm-l] Re: [obm-l] Re: [obm-l] Re: [obm-l] Re: [obm-l] Espaço métrico - topologia

2013-04-04 Por tôpico Bernardo Freitas Paulo da Costa
2013/4/4 Pedro Angelo pedro.fon...@gmail.com:
 Oi Bernardo

 Acho que esse buraco é parecido com um problema que teve aqui na lista
 que você resolveu, que em torno de cada um dos x_i a gente coloca uma
 bola de raio menor que epsilon = inf |x_n - x_m|, e aí dentro de cada
 uma dessas bolas a gente define f(x)=epsilon-d(x,x_i), e fora delas
 define f(x)=0.
É mais ou menos isso, no caso muito particular que nós estamos
tratando. Eu gostaria de ter um Teorema de Extensão Enumerável (ou
talvez até não-enumerável) sem precisar de continuidade uniforme, mas
com outro tipo de condição, para fazer isso de uma tacada só em f.

 Hmmm, só que não pode definir f(x)=0. Tem que pensar com mais calma mesmo.
Basta mudar a sua definição para que f(x) = 1 nas esferas d(x, x_i) = eps ;-)

Mas, como eu disse, é porque a gente tem informações demais sobre os
x_i. Talvez o único jeito seja esse mesmo, mas seria legal ter um
resultado um pouco mais geral, e sem depender demais do zero... o que
pode ser bem difícil porque a distância só dá números reais...

Abraços,
-- 
Bernardo Freitas Paulo da Costa

-- 
Esta mensagem foi verificada pelo sistema de antivírus e
 acredita-se estar livre de perigo.


=
Instruções para entrar na lista, sair da lista e usar a lista em
http://www.mat.puc-rio.br/~obmlistas/obm-l.html
=


[obm-l] Re: [obm-l] Re: [obm-l] Olimpíada regional (RJ)

2013-04-04 Por tôpico Bernardo Freitas Paulo da Costa
2013/4/4 saulo nilson saulo.nil...@gmail.com:
 a^p=amodp
 =760mod1998-20mod1998+1910mod1998-652mod1998=(760-20+1910-652)mod1998=1998mod1998
 =0mod1998
O único problema é que 1998 não é primo.
-- 
Bernardo Freitas Paulo da Costa

-- 
Esta mensagem foi verificada pelo sistema de antivírus e
 acredita-se estar livre de perigo.


=
Instruções para entrar na lista, sair da lista e usar a lista em
http://www.mat.puc-rio.br/~obmlistas/obm-l.html
=


Re: [obm-l] Limite

2013-04-03 Por tôpico Bernardo Freitas Paulo da Costa
2013/4/3 Heitor Bueno Ponchio Xavier heitor.iyp...@gmail.com:
 Galera, não consegui resolver a seguinte questão:
 Para todo r real, defina n(r)=#((m,n)∈ Z² | m²+n² r²)
 Calcule o limite:
 limite n(r)/r²r-infinito
Você tem que ver o que n(r) quer dizer, senão é impossível. Dica, a
resposta começa com 3 ;-)

-- 
Bernardo Freitas Paulo da Costa

-- 
Esta mensagem foi verificada pelo sistema de antiv�rus e
 acredita-se estar livre de perigo.


=
Instru��es para entrar na lista, sair da lista e usar a lista em
http://www.mat.puc-rio.br/~obmlistas/obm-l.html
=


[obm-l] Re: [obm-l] Mostrar que não existe f derivável tal que f(f(x )) = exp(-x^3)

2013-04-01 Por tôpico Bernardo Freitas Paulo da Costa
2013/3/30 Artur Costa Steiner steinerar...@gmail.com:
 Esta está na mesma linha de umas outras que enviei há uns 2 meses. Achei 
 interessante.

 Mostre que não existe f:R -- R diferenciável tal que f(f(x)) = exp(-x^3) 
 para todo real x.
Outra idéia:

Derive, como há dois meses atrás, e obtenha f'(f(x)) f'(x)  0 para x
diferente de 0, e f'(f(0)) f'(0) = 0. Assim, f'(x) != 0 para x != 0, e
pelo Teorema do valor intermediário para a derivada, f' tem sinal
constante em cada um dos intervalos (-infinito, 0) e (0, infinito).

Nova substitução: seja x = f(y).

f'(f(f(y))) f'(f(y))  0, e também f'(f(y)) f'(y)  0. Se f(y) != 0,
temos então que f'(y) tem o mesmo sinal de f'(f(f(y))) = f'(
exp(-y^3)). Note que isso vale para y negativo e y positivo. Note que
a derivada de f nunca é zero em R \ {0}, ou seja, em cada parte x  0
e x  0 sempre haverá pelo menos um y tal que f(y) seja diferente de
zero. Daí, vemos que f' tem o mesmo sinal tanto à esquerda como à
direita do zero, e daí f é monótona. Mas então f o f não pode ser
decrescente!

Aqui tanto o fato de exp(-x^3) ser decrescente como o de ser positiva
ajudaram. Se for f o f (x) = exp(-x^3) - 2, talvez tenha que estudar
melhor a função. (fica como problema para o próximo leitor)

Abraços,
-- 
Bernardo Freitas Paulo da Costa

-- 
Esta mensagem foi verificada pelo sistema de antivírus e
 acredita-se estar livre de perigo.


=
Instruções para entrar na lista, sair da lista e usar a lista em
http://www.mat.puc-rio.br/~obmlistas/obm-l.html
=


[obm-l] Re: [obm-l] Outra de composição de funções

2013-04-01 Por tôpico Bernardo Freitas Paulo da Costa
2013/4/1 Artur Costa Steiner steinerar...@gmail.com:
 Suponhamos que f :R -- R seja diferenciável e seja g = f o f. Mostre que, 
 se g for decrescente, então temos g'(x) = 0 para pelo menos 2 valores 
 distintos de x.

 Abraços.
Muito bom!

Se g' != 0, f' também, monótona, fim.

Se fosse um ponto só, f' tem sinal contrário em cada um dos intervalos
(-inf, x0), (x0, inf). (senão seria impossível fazer f'(f(x)) f'(x) 
0 !)
Mas então f(x) está do outro lado de x com relação a x0: f(x+)  x0,
f(x-)  x0.
Como f é contínua, f(x0) = x0.
Mas essas desigualdades e f(x0) = x0 dizem que f'(xi+) tem sinal
negativo para o xi dado pelo Teorema do Valor Médio para (x0, x+). A
mesma coisa vale para f'(xi-), que também seria negativo. Absurdo.

Você tem um exemplo com g'(x) = 0 apenas para dois valores de x?
-- 
Bernardo Freitas Paulo da Costa

-- 
Esta mensagem foi verificada pelo sistema de antivírus e
 acredita-se estar livre de perigo.


=
Instruções para entrar na lista, sair da lista e usar a lista em
http://www.mat.puc-rio.br/~obmlistas/obm-l.html
=


[obm-l] Re: [obm-l] Re: [obm-l] Mostrar que não existe f derivável tal que f(f(x )) = exp(-x^3)

2013-04-01 Por tôpico Bernardo Freitas Paulo da Costa
2013/4/1 Artur Costa Steiner steinerar...@gmail.com:
 É isso aí Bernardo.

 Eu desta vez dei a seguinte abordagem:

 Temos que g é derivável e decrescente. Suponhamos que a função f exista. Se 
 em algum intervalo I de R tivermos sempre f'(x) = 0, então f é crescente em 
 I e, desta forma, f o f = g, contrariamente ao que vimos, também é.
Foi rápido demais pra mim. O que acontece se f(I) = J, f é crescente
em I e decrescente em J? Como provar que existe um intervalo I tal que
f(I) está contido em I ?

-- 
Bernardo Freitas Paulo da Costa

-- 
Esta mensagem foi verificada pelo sistema de antivírus e
 acredita-se estar livre de perigo.


=
Instruções para entrar na lista, sair da lista e usar a lista em
http://www.mat.puc-rio.br/~obmlistas/obm-l.html
=


[obm-l] Re: [obm-l] Olimpíada regional (RJ)

2013-03-31 Por tôpico Bernardo Freitas Paulo da Costa
2013/3/31 marcone augusto araújo borges marconeborge...@hotmail.com:
 Prove que 760^1998 - 20^1998 + 1910^1998 - 652^1998 é divisivel por 1998

 Eu notei que 760 -20 + 1910 - 652 = 1998,mas...
Eu acho que vai ter que fatorar mesmo. 1998 = 2*999 = 2*9*111 =
2*9*3*37. Daí, é mandar Fermat em cada um dos fatores (claro que você
vai ignorar o 2) e depois juntar tudo com a | c e b | c e a,b primos
entre si = ab | c. Se não for isso, não sei o que é...

Abraços,
-- 
Bernardo Freitas Paulo da Costa

-- 
Esta mensagem foi verificada pelo sistema de antivírus e
 acredita-se estar livre de perigo.


=
Instruções para entrar na lista, sair da lista e usar a lista em
http://www.mat.puc-rio.br/~obmlistas/obm-l.html
=


Re: [obm-l] Desigualdades

2013-03-19 Por tôpico Bernardo Freitas Paulo da Costa
2013/3/19 Carlos Yuzo Shine cysh...@yahoo.com:
 Só para evitar derivadas (especialmente de mais de uma variável, em que há 
 vários detalhes), aí vão soluções:

 1) Pela desigualdade de médias, a expressão é igual a 4xy + (x^2 + 4y^2) + 
 2z^2 = 4xy + 4xy + 2z^2 = 8xy + 2z^2 = 4xyz = 4*32 = 128. A igualdade 
 ocorre quando x = 2y e 4xy = z^2, ou seja, x = 2^(11/6), y = 2^(5/6) e z = 
 2^(7/3).

Oi Shine,

eu não entendi a passagem 8xy + 2z^2 = 4xyz. Não pode ser só
desigualdade das médias, porque essa é homogênea, e todos os termos da
esquerda são de ordem dois. Acho que faltou uma dica para o seu caro
leitor.

Pensando um pouco mais, eu resolveria com multiplicadores de Lagrange
(e portanto com derivadas). Mas se fosse antes de aprender Lagrange,
eu teria feito assim:

Note que se z é fixo, temos que minimizar (x + 2y)^2, com xy =
constante. (Aplicando a famosa técnica escolha produtos notáveis que
vão te ajudar.) Pela MA = MG, obtemos x = 2y (como todo mundo
obteve...).

xy = 32/z, x = 2y = 2y^2 = 32/z = y^2 = 16/z, x^2 = 4*16/z e
portanto x^2 + 4xy + 4y^2 = 4*16/z + 4*32/z + 4*16/z = 4*32*2/z.

Queremos minimizar 4*32*2/z + 2z^2. Pela desigualdade das médias com 3
termos: 4*32/z + 4*32/z + 2z^2 = 3 * (4*32 * 4*32 * 2)^1/3 = 3 *
(2^(2+5+2+5+1))^1/3 = 3 * 2^5 = 3 * 32 = 96. A igualdade ocorre para

4*32/z = 2z^2 = 64 = z^3, ou seja z = 4, y = 4/raiz(z) = 4/2 = 2, x = 4.

Verificando: x^2 = 4^2 = 16
4xy = 4*2*4 = 32
4*y^2 = 4*2^2 = 16
2z^2 = 2*4^2 = 32
Somando = 96.

-- 
Bernardo Freitas Paulo da Costa

=
Instruções para entrar na lista, sair da lista e usar a lista em
http://www.mat.puc-rio.br/~obmlistas/obm-l.html
=


[obm-l] Re: [obm-l] Número composto

2013-03-16 Por tôpico Bernardo Freitas Paulo da Costa
2013/3/16 marcone augusto araújo borges marconeborge...@hotmail.com:
 Mostre que para todo inteiro a  1,existe um primo p tal que 1 +  a + a^2 +
 ...+ a^(p-1) é composto.

Veja que este problema é bem fácil para metade dos a's. Se a é ímpar,
1+a é par, que é composto, p = 2 serve. Tente estender para a = 2, 4,
6, usando o menor número possível de termos a cada vez.

Abraços,
-- 
Bernardo Freitas Paulo da Costa

=
Instruções para entrar na lista, sair da lista e usar a lista em
http://www.mat.puc-rio.br/~obmlistas/obm-l.html
=


Re: [obm-l] Geometria

2013-03-13 Por tôpico Bernardo Freitas Paulo da Costa
2013/3/13 marcone augusto araújo borges marconeborge...@hotmail.com:
 Saja um triângulo cujos lados medem a,b e c e R o raio da circunferência
 circunscrita.
 Mostre que a^2 + b^2 + c^2 = 8R^2 se,e somente se,o triângulo é retângulo.

 Se o triangulo é retangulo,considerando a  = b  c,temos que a^2 + b^2 =
 c^2
 a^2 + b^2 + c^2= 2c^2 e,como c = 2R,segue que
 a^2 + b^2 + c^2 = 2.(2R)^2 = 8R^2
 Estou tentando a segunda parte da demonstração e não sai.
Eu fiz na marra... Chame os ângulos internos do seu triângulo de A, B,
C. Portanto, a = 2R sin(A), b = 2R sin(B), c = 2R sin(C). Note que C =
pi - A - B = sin(C) = sin(A+B).

Eleve tudo ao quadrado, cancele 4 R^2, fica:
sin^2(A) + sin^2(B) + sin^2(A+B) = 2

1 - sin^2 = cos^2, logo passando os senos pro outro lado

sin^2(A+B) = cos^2(A) + cos^2(B)

Mas também sin(A+B) = sin(A) cos(B) + sin(B) cos(A), ao quadrado temos

sin^2(A) cos^2(B) + 2 sin(A) cos(B) sin(B) cos(A) + sin^2(B) cos^2(A)
= cos^2(A) + cos^2(B)

Passe de novo termos pro outro lado:

2 sin(A) cos(B) sin(B) cos(A) = cos^2(A) (1 - sin^2(B)) + cos^2(B) (1
- sin^2(A)) = 2 cos^2(A) cos^2(B)

Cancele os cossenos

sin(A) sin(B) = cos(A) cos(B), ou então cos(A + B) = 0. Como os
ângulos estão entre 0 e pi/2 (SPG C é o maior ângulo) a única solução
é A + B = pi/2.

Deve dar para fazer por desigualdades também...
-- 
Bernardo Freitas Paulo da Costa

-- 
Esta mensagem foi verificada pelo sistema de antivírus e
 acredita-se estar livre de perigo.


=
Instruções para entrar na lista, sair da lista e usar a lista em
http://www.mat.puc-rio.br/~obmlistas/obm-l.html
=


[obm-l] Re: [obm-l] Séries

2013-03-02 Por tôpico Bernardo Freitas Paulo da Costa
2013/3/1 Artur Costa Steiner steinerar...@gmail.com:
 Acho estes interessantes

 Seja a_n é uma sequencia de reais positivos e s_n a sequência de suas somas 
 parciais. Mostre que as seguintes séries convergem se, e somente se, s_n 
 converge.

 1) Soma (a_n)/(s_n)
Muito bom esse critério! Eu só conseguir fazer porque eu roubei e
supus que a_n fosse monótona, daí você usa comparação integral-soma,
e daí eu vi qual era a fórmula. Mais um pouquinho de esforço eu achei
a transformação certa. Sem isso eu nunca teria acreditado que era (x ~
- log(1-x)) e não o mais habitual x ~ log(1+x). Mas talvez seja porque
eu estou no século errado, há um tempo todo mundo faria x = - log(1-x)
*antes* de pensar na log(1+x). Aliás, vendo tantos logs, faz pensar um
pouco no critério de condensação de Cauchy também.

 2) Soma (a_n)/(a_n + k), k  0
Essa eu achei mais fácil ;-)

-- 
Bernardo Freitas Paulo da Costa

-- 
Esta mensagem foi verificada pelo sistema de antivírus e
 acredita-se estar livre de perigo.


=
Instruções para entrar na lista, sair da lista e usar a lista em
http://www.mat.puc-rio.br/~obmlistas/obm-l.html
=


[obm-l] Re: [obm-l] Questões interessantes (na minha opinião)

2013-03-02 Por tôpico Bernardo Freitas Paulo da Costa
2013/3/1 Artur Costa Steiner steinerar...@gmail.com:
 1) suponhamos que exista uma função f tal que, para todo real x, tenhamos 
 f(f(x)) = ax^2 + bx + c, a não nulo, b e c reais. Mostre que (b +1)(b - 3) = 
 4ac.
Esse eu ainda tenho que pensar com cuidado. A primeira coisa é reduzir
a g(g(x)) = x^2 + c, mas eu ainda não sei fazer o caso c  0.

 2) seja (a_n) uma sequência de reais e (p_n) uma sequência de pesos 
 positivos. Seja (s_n) a sequência das médias ponderadas de (a_n) com relação 
 os pesos p_n. Mostre que, se Soma p_n divergir, então

 liminf s_n = liminf a_n = limsup a_n = limsup s_n

 É bem fácil mostrar que, se Soma p_n convergir, as desigualadas da direita e 
 da esquerda não têm que valer.
Curioso... Eu diria que s_n é uma combinação convexa dos a_n, logo s_N
= min(a_n, n=1..N) e portanto min(s_N, N=1..k) = min(min(a_n,
n=1..N), N=1..k) = min(a_n, n=1..k). Claro que tem que fazer do outro
lado (no infinito, não no 1) mas eu diria que liminf a_n = liminf
s_n. Mais tarde tento enviar uma prova dessa soma de Césaro.

 3) Seja f uma função definida em um intervalo I de R (suponhamos aberto, para 
 facilitar) e com valores em R. Suponhamos que, em cada ponto de I, as 4 
 derivadas de Dini de f existam e sejam finitas. Mostre que existe um 
 subintervalo de I no qual f é Lipschitz.
 Isto é mais fácil de mostrar se supusermos diferenciabilidade cheia em todo o 
 I. Mas, de fato, basta a existência das 4 derivadas de Dini.
Diferenciabilidade cheia = full differentiability = diferenciável no
sentido usual ? (Nunca fiz nada com derivadas de Dini)

-- 
Bernardo Freitas Paulo da Costa

-- 
Esta mensagem foi verificada pelo sistema de antivírus e
 acredita-se estar livre de perigo.


=
Instruções para entrar na lista, sair da lista e usar a lista em
http://www.mat.puc-rio.br/~obmlistas/obm-l.html
=


Re: [obm-l] Integral interessante

2013-03-01 Por tôpico Bernardo Freitas Paulo da Costa
2013/2/24 Artur Costa Steiner steinerar...@gmail.com:
 Seja f uma função real ímpar, contínua em toda a reta real. Seja a  0. 
 Determine Int[-a, a] 1/(e^(f(x)) + 1) dx

Se eu não errei as contas, I = a. Para conferir, f(x) = 0, dá certo.

Abraços,
-- 
Bernardo Freitas Paulo da Costa

-- 
Esta mensagem foi verificada pelo sistema de antivírus e
 acredita-se estar livre de perigo.


=
Instruções para entrar na lista, sair da lista e usar a lista em
http://www.mat.puc-rio.br/~obmlistas/obm-l.html
=


Re: [obm-l] Pecinhas

2013-02-12 Por tôpico Bernardo Freitas Paulo da Costa
2013/2/12 Pedro Nascimento pedromn...@gmail.com:
 Nao pensei muito , mas acho que a ideia eh montar uma recorrencia definindo
 os possiveis inicios na forma de colocar as pecas , voce define
 possibilidades disjuntas no modo como distribuir as pecas. Os possiveis
 inicio sao:

 QL
 LL

 LQ
 LL

 LL
 LQ

 LL
 QL

 Q
 Q

 LLL
 LLL

 onde L indica um pedaco de um L e Q um quadrado, definindo esses
 inicios, podemos distribuir o  sobrou de forma recorrente. Assim ficamos
 com a seguinte recorrencia:

 F(N)=F(N-3)+4*F(N-2)+F(N-1) para N=3

 com os casos base. F(0)=1 , F(1)=1 , F(2)=4
Eu botei a característica no wolfram alfa, e saiu um treco muito
engraçado: todas as raízes são complexas. Parece que o nosso wolfram
ainda não sabe que um polinômio de grau ímpar sempre tem uma raiz
real. http://www.wolframalpha.com/input/?i=x^3+-+x^2+-+4*x+-+1+%3D+0

E também diz que a solução vai ser feia...
-- 
Bernardo Freitas Paulo da Costa

=
Instruções para entrar na lista, sair da lista e usar a lista em
http://www.mat.puc-rio.br/~obmlistas/obm-l.html
=


[obm-l] Re: [obm-l] Desigualdade envolvendo números primos

2013-02-11 Por tôpico Bernardo Freitas Paulo da Costa
2013/2/11 Artur Costa Steiner artur_stei...@yahoo.com:
 Seja (q(n)) = (2, 5, 11, 17...) a sequencia que enumera os primos de ordem 
 ímpar. Isto é, q(n) = primo de ordem 2n - 1.

 Mostre que, para todo k  1, a desigualdade q(n)  n^k ocorre para uma 
 infinidade de valores de n.
Vale usar o TNP?


-- 
Bernardo Freitas Paulo da Costa

=
Instruções para entrar na lista, sair da lista e usar a lista em
http://www.mat.puc-rio.br/~obmlistas/obm-l.html
=


[obm-l] Re: [obm-l] Re: [obm-l] Re: [obm-l] RE: [obm-l] Re: [obm-l] Sequências de Funções

2013-02-10 Por tôpico Bernardo Freitas Paulo da Costa
2013/2/10 Artur Costa Steiner steinerar...@gmail.com:
 Estes dois livros são excelentes. Tem também o do Zrudin eo do Apostol.
Zrudin é porque ele usa variáveis complexas?

-- 
Bernardo Freitas Paulo da Costa

=
Instruções para entrar na lista, sair da lista e usar a lista em
http://www.mat.puc-rio.br/~obmlistas/obm-l.html
=


[obm-l] Re: [obm-l] RE: [obm-l] Número racional

2013-02-08 Por tôpico Bernardo Freitas Paulo da Costa
2013/2/8 João Maldonado joao_maldona...@hotmail.com
 From: marconeborge...@hotmail.com
  Determine todos os inteiros positivos a e b para os quais o número
  (raiz(2) + raiz(a))/(raiz(3) + raiz(b)) é racional

 (raiz(2) + raiz(a))/(raiz(3) + raiz(b)) = racional
 ENTÃO[ (2+a) + 2raiz(a)]/[(3+b) + 2 raiz(3b)] também é racional
 Mas a volta não vale, já que nem todo número é quadrado perfeito

Confesso que ainda não tive tempo de ler com cuidado sua solução. Mas
(a,b) = (3,2) com certeza é uma solução, e eu não vi no seu scan.

Abraços.
--
Bernardo Freitas Paulo da Costa

=
Instruções para entrar na lista, sair da lista e usar a lista em
http://www.mat.puc-rio.br/~obmlistas/obm-l.html
=


[obm-l] Re: [obm-l] Sequências de Funções

2013-02-07 Por tôpico Bernardo Freitas Paulo da Costa
2013/2/7 Sandoel Vieira sandoe...@hotmail.com:
 Mostre que não existe uma sequências de funções contínuas f_n:[0,1]--R,
 convergindo simplesmente para a função f:[0,1]--R tal que f(x)=0 para x
 racional e f(x)=1 quando x é irracional.
Pense no que acontece para que f_n(1/2) - 0, e nos pontos da
vizinhança de 1/2 para este n fixo. Agora, pense nos outros pontos
pontos racionais perto de 1/2, e repita o argumento. Agora, lembre que
os racionais são densos, e que você tem um monte de vizinhanças em
todos os pontos racionais.
-- 
Bernardo Freitas Paulo da Costa

=
Instruções para entrar na lista, sair da lista e usar a lista em
http://www.mat.puc-rio.br/~obmlistas/obm-l.html
=


Re: [obm-l] Desigualdade

2013-02-05 Por tôpico Bernardo Freitas Paulo da Costa
2013/2/5 marcone augusto araújo borges marconeborge...@hotmail.com:
 9(a^3 +b^3 + c^3)  = (a + b + c)^3
 Usando (a+b+c)^3 = a^3 + b^3 +c^3 + 3(a + b)(a + c)(b+c),basta
 mostrar que 8(a^3 + b^3 + c^3)  = 3(a+b)(a+c)(b+c)
Está faltando uma carta na sua manga:
http://en.wikipedia.org/wiki/Muirhead%27s_inequality.
-- 
Bernardo Freitas Paulo da Costa

=
Instruções para entrar na lista, sair da lista e usar a lista em
http://www.mat.puc-rio.br/~obmlistas/obm-l.html
=


[obm-l] Re: [obm-l] Convergência de somas de Riemann

2013-01-30 Por tôpico Bernardo Freitas Paulo da Costa
2013/1/30 Artur Costa Steiner steinerar...@gmail.com:
 Eu dei uma prova para isto, mas acho que só vale para funções não negativas.

 Seja f uma função definida e contínua no intervalo finito (a, b] tal que sua 
 integral imprópria sobre este intervalo exista e seja finita (como f(x) = 
 1/raiz(x) em (0, 1], que, no caso, vai para infinito em 0+). Seja (P_n) uma 
 sequência de partições de [a, b] tal que ||P_n|| -- 0 e (S_n) uma sequência 
 de somas de Riemann de f sobre [a, b] tal que, em cada intervalo de cada P_n, 
 f seja tomada no ponto em que, no dado intervalo, f apresente seu valor 
 mínimo (a continuidade de f garante a existência destes pontos). Mostre que 
 S_n -- Int (a, b] f(x) dx, integral imprópria.

Oi Artur,

o problema é que f seja tomada no ponto em que, no dado intervalo, f
apresente seu valor mínimo está mal-definido no intervalo da partição
contendo a extremidade a, se porventura f - -infinito em x - a+.
Claro, aposto que isso é irrelevante para a integral, já que, sendo
justamente convergente em a, acho que o peso da cauda infinita em
a é zero. Assim, talvez o que você quer seja que || P_n || - 0 e você
começa as somas de Riemann apenas a partir do segundo intervalo. Se o
que eu falei está certo, acho que não precisa considerar que f seja
contínua: basta saber que toda soma de Riemann associada a P_n será
maior do que a associada à função constante por partes e que vale o
ínfimo de f em cada um dos segmentos, excluído o primeiro, que não
entra na história.

Abraços,
-- 
Bernardo Freitas Paulo da Costa

=
Instruções para entrar na lista, sair da lista e usar a lista em
http://www.mat.puc-rio.br/~obmlistas/obm-l.html
=


[obm-l] Re: [obm-l] Re: [obm-l] Espaço métrico compacto

2013-01-17 Por tôpico Bernardo Freitas Paulo da Costa
2013/1/17 Artur Costa Steiner steinerar...@gmail.com:
 Considere as seguintes funções (de Uryson) : g_n = 1 em x_n, g_n(x) =
 0 para d(x,x_n)  r/3.

 Estou com uma dúvida. Qual a definição de g_n(x) se 0  d(x, x_n) = r/3?

Opa, foi rápido demais. As g_n são dadas pelo TET / Teorema de Uryson,
que garantem que você pode fazer g_n contínua valendo 1 num ponto e
zero fora de uma vizinhança mais longe. Eu quis escrever assim para
ficar mais fácil de tentar continuar no caso em que K não fosse
composto apenas por pontos isolados, mas não deu tempo ontem.

Abraços,
-- 
Bernardo Freitas Paulo da Costa

=
Instruções para entrar na lista, sair da lista e usar a lista em
http://www.mat.puc-rio.br/~obmlistas/obm-l.html
=


[obm-l] Re: [obm-l] Re: [obm-l] Convergência uniforme

2013-01-08 Por tôpico Bernardo Freitas Paulo da Costa
2013/1/8 Artur Costa Steiner steinerar...@gmail.com:
 Legal Bernardo!

 Vc está dizendo que se f_n é uma sequência de holomorfas, uniformemente 
 limitadas por um M em um compacto K, que convirja neste conjunto para uma 
 função f, então a convergência é uniforme? É essa a idéia?
Isso mesmo. Escreva o que aconteceria se a convergência não fosse
uniforme (uma seqüência x_n vai aparecer, o que é bom porque K é
compacto). Agora, tente lembrar dos contra-exemplos clássicos de
convergência uniforme e veja que uma derivada vai explodir. Mas a
fórmula de Cauchy garante que, longe do bordo de K, a derivada não
pode explodir.

Abraços,
-- 
Bernardo Freitas Paulo da Costa

=
Instruções para entrar na lista, sair da lista e usar a lista em
http://www.mat.puc-rio.br/~obmlistas/obm-l.html
=


[obm-l] Re: [obm-l] Funções polinomiais e exponenciais nos complexos

2013-01-06 Por tôpico Bernardo Freitas Paulo da Costa
2012/12/12 Artur Costa Steiner steinerar...@gmail.com:
 Seja P um polinômio complexo não constante e sejam k e a constantes complexas 
 não nulas. Mostre que

 1) a equação P(z) = k exp(az) tem uma infinidade de raízes

 2) em toda reta do plano complexo, a equação acima tem um número finito de 
 raízes.

Antes de dar uma resposta completa, vou dar uma idéia só: Newton e
periodicidade. Ah, sim, SPG, k = 1 e a = 1 também, mudando as
variáveis, mas não é realmente útil no argumento, é mais pra limpar a
notação.
-- 
Bernardo Freitas Paulo da Costa

=
Instruções para entrar na lista, sair da lista e usar a lista em
http://www.mat.puc-rio.br/~obmlistas/obm-l.html
=


[obm-l] Re: [obm-l] Re: [obm-l] Mostrar que não existe funcões diferenciáveis de R em R que satisfaçam a estas condições

2012-12-16 Por tôpico Bernardo Freitas Paulo da Costa
2012/12/16 Artur Costa Steiner steinerar...@gmail.com:
 f(f(x)) = cos(x)

 Antes, consideremos os seguintes lemas

 1) seja f definida no domínio D e com valores em D e seja g = f o f. Se a em 
 D for o único ponto fixo de g em D, então é ponto fixo de f (também o único, 
 pois todo ponto fixo de f é ponto fixo de g)

 Prova.: seja b = f(a). Então, f(b) = f(f(a)) = g(a) = a e, portanto, g(b) = 
 f(f(b)) = f(a) = b, o que mostra que b é ponto fixo de g. Mas como g tem um 
 único ponto fixo em a, segue-se que b = a, de modo que a = f(a). Logo, a é 
 ponto fixo de f.

Muito bem! Mas você usa esse lema com D = R, porque nada se sabe de f
a priori, e eu achei estranho você usar o D. Mas, claro, está certo, e
as hipóteses estão perfeitas: o que é importante é que f(D) esteja
contido em D, porque se f for longe e voltar depois não dá pra
controlar. Por isso mesmo que eu (sem saber da sua demonstração)
também comecei provando que f(0) não ia longe demais.

 2) se f de R em R é diferenciável e apresenta um ponto fixo em a, então g = f 
 o f é diferenciável em a e g'(a) = (f'(a))^2.

 Prova.: da regra da cadeia, segue-se que f é diferenciável em a e que g'(a) = 
 f'(f(a)) f'(a) = f'(a) f'(a) = (f'(a))^2

 Disto é imediato que g'(a) = 0.

 Definamos h(x) = x - cos(x). Verificamos que h só pode se anular em [0, 1]. 
 Neste intervalo, h'(x) = 1 + sen(x)  0, de modo que h é estritamente 
 crescente, logo bijetora. Como pi/2 está em [0, 1], h(0) = -1 e h(pi/2) = 
 pi/2, concluímos que, em todo R, g(x) = cos(x) tem um único ponto fixo em 
 algum a de (0, pi/2). Como g'(a) = -sen(a)  0, segue-se dos lemas 1 e 2 que 
 não existe f diferenciável com f(f(x)) = cos(x) para todo x.

 Generalização. Se g é diferenciável e tem um único ponto fixo em a de R com 
 g'(a)  0, então não há f diferenciável com f o f = g

Só pra ter certeza, esse é, então, completamente diferente dos
anteriores. Porque basta que g seja diferenciável *exatamente* no
ponto fixo, e f também. Por exemplo, não existe f diferenciável em 0
tal que f o f = -sin(x). Claro que ainda tem a idéia do g'  0, mas é
independente de Darboux, e uma idéia mais dinâmica.

 Abraços
 Artur

Abraços,
-- 
Bernardo Freitas Paulo da Costa

=
Instruções para entrar na lista, sair da lista e usar a lista em
http://www.mat.puc-rio.br/~obmlistas/obm-l.html
=


[obm-l] Re: [obm-l] Re: Mostrar que não existe funcões diferenciáveis de R em R que satisfaçam a estas condições

2012-12-10 Por tôpico Bernardo Freitas Paulo da Costa
2012/12/10 Artur Costa Steiner steinerar...@gmail.com:
 Não sei de onde sairam estes caracteres malucos

 Sim, claro, são 3 questões distintas. Nem é possível atender duas delas
 simultaneamente.
Muito legais! Como você os encontrou? Ah, está em ordem de dificuldade?
-- 
Bernardo Freitas Paulo da Costa

=
Instruções para entrar na lista, sair da lista e usar a lista em
http://www.mat.puc-rio.br/~obmlistas/obm-l.html
=


[obm-l] Re: [obm-l] ajuda em exercício de desigualdade

2012-12-01 Por tôpico Bernardo Freitas Paulo da Costa
2012/12/1 Bruno Rodrigues bruninhu_1...@hotmail.com:
 Olá galera,estou travado nesse problema que segue:

 Ache o maior valor inteiro positivo de n tal que:
  n^²°°5^³°°
Eu imagino que seja n^200  5^300. (Dica: EVITE qualquer coisa que não
seja letras e números normais.)

Bom, eu faria no braço: logaritmo dos dois lados, fica
2*log(n)  3*log(5) = n^2  125, portanto n = 11.
-- 
Bernardo Freitas Paulo da Costa

=
Instruções para entrar na lista, sair da lista e usar a lista em
http://www.mat.puc-rio.br/~obmlistas/obm-l.html
=


[obm-l] Re: [obm-l] Equações(inteiros)

2012-11-23 Por tôpico Bernardo Freitas Paulo da Costa
2012/11/23 marcone augusto araújo borges marconeborge...@hotmail.com:
 Como resolver as equações ?

 1) x(y+1)^2 = 243y
Use que, em geral, y+1 é primo com y.

 2) 1/a + 1/b + 1/c = 1
No braço. Ordene a = b = c, e tente ver que c não pode ser muito grande.

 3) x^3 + 21y + 5 = 0
Sei lá. Você quer que 21(-y) seja x^3 + 5. Eu faria uma tabela com os
restos mod 21 dos cubos, e veria se tem algum = 5. Você pode fazer por
3 e 7 separado, lembrando que 5 = 2 mod 3 e 5 mod 7. Deve dar um
absurdo, ou então uma equação modular para x (tipo x congruente a ...
módulo 21, ou seja, x = 21m + n) e daí, como o 21y é linear, todos os
x dessa forma vão dar uma solução em y.

Abraços,
-- 
Bernardo

=
Instruções para entrar na lista, sair da lista e usar a lista em
http://www.mat.puc-rio.br/~obmlistas/obm-l.html
=


[obm-l] Re: [obm-l] Re: [obm-l] Mostrar que P não tem raízes com ambas as partes racionais

2012-11-21 Por tôpico Bernardo Freitas Paulo da Costa
 Em 21 de novembro de 2012 19:50, Artur Costa Steiner
 steinerar...@gmail.com escreveu:

  Gostaria de propor esta demonstração
 
  Seja P um polinômio com coeficientes inteiros tal que (1) o coeficiente
  do termo líder e o termo independente são ímpares e (2) o número total de
  coeficientes ímpares é ímpar. Como em
 
  P(x) = x^4 - 5x^3 + 4x^2 - 6x - 3
  Q(x) = 5x^3 +  8x^2 - 3x + 7
  R(x) = x^700 - 17x^423 + 13
 
  Mostre que P não tem nenhuma raiz em que as partes real e imaginária
  sejam ambas racionais.

2012/11/21 terence thirteen peterdirich...@gmail.com:
 Módulo 2?
Além disso, eu precisei de duas coisas:
- para n fixo, os números binomiais (n, k) que são ímpares são sempre por pares;
- (1+i) = raiz(2) exp(pi * i / 4).

-- 
Bernardo Freitas Paulo da Costa

=
Instruções para entrar na lista, sair da lista e usar a lista em
http://www.mat.puc-rio.br/~obmlistas/obm-l.html
=


[obm-l] Re: [obm-l] Espaços

2012-11-18 Por tôpico Bernardo Freitas Paulo da Costa
2012/11/17 Athos Couto athos...@hotmail.com:
 Boa tarde pessoal.
 Rn-1 está contido em Rn?

 Caso a resposta seja sim, por que Rn-1 não é um subespaço de Rn?
Como você mesmo provou, R^(n-1) não está contido em R^n.

O grande problema é que em geral a gente considera que o subespaço
(x1, x2, ..., x(n-1), 0) é igual a R^(n-1), e com isso diz que R^1
está contido em R^2, etc..
--
Bernardo Freitas Paulo da Costa

=
Instruções para entrar na lista, sair da lista e usar a lista em
http://www.mat.puc-rio.br/~obmlistas/obm-l.html
=


Re: [obm-l] OBM 2011

2012-10-13 Por tôpico Bernardo Freitas Paulo da Costa
 Pensando aqui, 3^2+4^2=25, logo 34 é chapa. Não deve ser difícil ver
 que 34344433 é chapa, pois a soma é 4*(3^2+4^2).

 Assim, para todo N múltiplo de 8, existe um chapa de N algarismos.
 Difícil vai ser cobrir os restantes... :(
Eu sei fazer com 4, 5, 6, e 7 algarismos, roubando a idéia do Torres.
Só não sei como ele faz com 16 algarismos, já que 8*(3^2 + 4^2) não é
um quadrado... Você provou que dá pra fazer todos os 8N^2, na verdade,
com 3 e 4, você faz todos os 2n^2 com o mesmo argumento. Usando que
13^2 = 5^2 + 12^2 (12 = 2*6, ou 3*4, ou 4*3, ou 6*2) você deve
conseguir um monte de outros alfa*n^2. Mas o problema é que quadrados
são mutio raros nos números inteiros...
-- 
Bernardo Freitas Paulo da Costa

=
Instruções para entrar na lista, sair da lista e usar a lista em
http://www.mat.puc-rio.br/~obmlistas/obm-l.html
=


Re: [obm-l] OBM 2011

2012-10-13 Por tôpico Bernardo Freitas Paulo da Costa
2012/10/13 terence thirteen peterdirich...@gmail.com:
 Eu pensei em alguma indução, mas fala sério, tem que somar com alguma
 propriedade legal.

 Se pudéssemos fazer algo com ALPHA*m^2+BETA*n^2, em que m e n são
 primos entre si, um teorema de Bezout nos afirma que todo natural
 grande pode ser escrito como somas de vários  m^2 e n^2.
Que tal contar?

Entre ... e ... as somas dos quadrados dos dígitos variam de n
a 81n. Por outro lado, as somas possíveis são 9^n / 9! (descontando a
ordem). Logo deve haver um monte que coincidem. Mas acho que, com um
pouquinho de sorte, para n suficientement grade, temos praticamente
todos os valores possíveis entre n e 81n. Deve ter um número aí no
meio que seja um quadrado. Por exemplo, ([sqrt(n)] + 1)^2 é com
certeza menor do que 4n para n  1.

Chutando com um computador: para n suficientemente grande, todos os
números entre n+14 e 64n são factíveis. Provavelmente deve ser até
melhor do que isso no upper bound. O lower bound é mais fácil: você
tem um monte de 1. Trocar 1 por 2 aumenta três, 1 por 3 aumenta 8.
Fazer n+14 = n+8+3+3. n+15 é trocar 1 por 4. Fazer n+13 não dá, porque
as combinações com 8 e 3 não permitem. Mas o real problema é achar um
quadrado perto de n, o mais próximo pode ser ainda bm longe.
Imagine n = 1^2 + 1. O próximo está a 2*sqrt(n) de distância...
hum, e você pode somar 8 ou 3, e se sqrt(n) é grande o suficiente,
Bézout, acabou. (Você tem n casas para alterar, e 8 e 3 são maoires do
que 2, e n  sqrt(n)). O único caso ruim em que o próximo quadrado é
justamente n+13 (que não podemos fazer), o quadrado seguinte está a
2*sqrt(n+13) + 1 de distância, e o Bézout garante que podemos fazer
todas os inteiros entre 7*2 e 8*n - 7*2 (ou algo próximo a isso), se
tivermos no máximo n termos para escolher entre 3 e 8, e o crescimento
linear é mais do que suficiente. Agora, basta provar para os casos em
que n é pequeno, mas a gente já fez!

Quem se aventura a provar que dá pra fazer quase todos os números? Eu
aposto que tem a ver com a^2 + b^2 = (a-1)^2 + (b+1)^2 + 2(a-b-1).

Abraços,
-- 
Bernardo Freitas Paulo da Costa

=
Instruções para entrar na lista, sair da lista e usar a lista em
http://www.mat.puc-rio.br/~obmlistas/obm-l.html
=


Re: [obm-l] AB = I implica BA = I

2012-10-10 Por tôpico Bernardo Freitas Paulo da Costa
2012/10/10 Daniel Estrela destr...@gmail.com:
 Seja AB=I.
 Agora tome BI = B

 BI = B
 B(AB) = B
 (BA)B = B
 B - (BA)B = 0
 (I - BA)B = 0

 Como B é diferente de 0, então BA = I
A lei do corte não vale para matrizes. Por exemplo,
[0 0] x [1 0] = [0 0]
[0 1]   [0 0]   [0 0]

Aliás, isso dá mesmo um exemplo: seja A = identidade, B = [1 0; 0 0],
temos BAB = B, mas BA != Identidade.
-- 
Bernardo Freitas Paulo da Costa

=
Instruções para entrar na lista, sair da lista e usar a lista em
http://www.mat.puc-rio.br/~obmlistas/obm-l.html
=


Re: [obm-l] AB = I implica BA = I

2012-10-09 Por tôpico Bernardo Freitas Paulo da Costa
2012/10/9 Hugo Fernando Marques Fernandes hfernande...@gmail.com:
 Multiplique os dois lados da igualdade AB = I por B^(-1) (inversa de B) à
 direita e depois por B à esquerda...

 BAB(B^(-1)) = BI(B^(-1)) = BAI = BB^(-1) = BA = I

Vou ser chato (de novo). Em geral, quando se pede para mostrar que AB
= I = BA = I, é justamente para mostrar que a inversa funciona dos
dois lados. Daí (usando a sua notação) sabemos que B tem uma inversa à
esquerda que é A, e A tem uma inversa à direita que é B. Portanto,
ainda não sabemos que existe B^(-1) para multiplicar à direita de B.

O jeito que eu prefiro pra essa propriedade é ver que a matriz produto
de transformações elementares E que leva B na Identidade, leva a
Identidade em A. Isso dá duas igualdades para você:
E*B = I
E*I = A

A segunda diz que E = A, logo AB = I, que é daonde começa o problema do ennius.

Mas como você usou o algoritmo de Gauss para levar A na identidade, o
que acontece é que na parte da solução estão os vetores tais que B*v_i
= e_i. Essa outra parte mostra que BA = I, e portanto A é a inversa de
B.

Alguém sabe fazer de outra forma, sem apelar para matrizes?
-- 
Bernardo Freitas Paulo da Costa

=
Instruções para entrar na lista, sair da lista e usar a lista em
http://www.mat.puc-rio.br/~obmlistas/obm-l.html
=


Re: [obm-l] AB = I implica BA = I

2012-10-09 Por tôpico Bernardo Freitas Paulo da Costa
2012/10/9 Paulo Argolo pauloarg...@outlook.com:
 Usando-se determinantes:

 det(A.B) = det (A). det(B)= det(I) = 1
 Portanto, det(A) e det(B) são diferentes de zero. Logo, A e B são
 inversíveis.
 Sejam A' e B' as inversas de A e B, respectivamente.
 Então:
 A.B = I = A'.(A.B.) = A'.I = (A'.A).B = A' = I.B = A' = B=A' = B.A =
 A'.A
 = B.A = I
 Espero que esteja correto.
Hum, certo está, mas (mais uma vez) o grande problema é que você já
admite que existem inversas bilaterais. Bem ali quando você diz A e B
são inversíveis, já que a definição de inversíveis é justamente que
para uma matriz B, existe A tal que AB = I = BA. Claro que a maior
parte desses exercícios é apenas manipulação algébrica, e daí a
primeira demonstração é suficiente.

Para ser mais positivo: essa manipulação é um truque importante,
porque ela mostra que, se AB = I e se BC = I então A = C. Logo, se
existir uma inversa à esquerda e uma inversa à direita, elas são
iguais, e dá a inversa que você quer:
AB = I, multiplique por C, (AB)C = C, associativa, A(BC) = C = A = C

Mas eu não lembro de nada que diga que se existe uma inversa de um
lado, então existe uma inversa do outro, a não ser o argumento de
redução por operações elementares que eu falei. Vou tentar achar o
Hoffman  Kunze amanhã...
-- 
Bernardo Freitas Paulo da Costa

=
Instruções para entrar na lista, sair da lista e usar a lista em
http://www.mat.puc-rio.br/~obmlistas/obm-l.html
=


[obm-l] Re: [obm-l] Re: [obm-l] Re: [obm-l] Preciso de uma Solução ensino médio

2012-10-03 Por tôpico Bernardo Freitas Paulo da Costa
2012/10/3 terence thirteen peterdirich...@gmail.com:
 On Tue, 2 Oct 2012 23:13:27 -0300, terence thirteen wrote:
 Como seria essa torre? Simplesmente um empilhamento de todos os
 tijolos? Por exemplo, se eu empilhar algumas com a face de medida 4x10
 para o chão, mas deixar apenas uma com a face

 Assim, seria o total de valores de 4a+10b+19c, com a restrição a+b+c =
 94. O máximo seria 19*94 e o mínimo 4*94. Agora tem que ver quais não
 podem ser representados neste esquema.

 Assim sendo, eu teria que verificar.

 Fazendo c=0, 4a+10b pode assumir todos os pares acima de 6, ou seja,
 de 8 em diante. Somando 19, se obteriam todos  os ímpares de 27 em
 diante. Basta testar os ímpares abaixo de 27 e todos abaixo de 6...
O problema é a condição a + b + c = 94, e não a + b + c = 94. Assim,
o menor número que você pode obter é 4 * 94, mas o seguinte é 4 * 93 +
10 * 1, o seguinte 4 * 92 + 10 * 2, o seguinte 4 * 93 + 17 * 1 (sim, é
um pouquinho mais fácil porque 2 * 10 + 1 = 4 + 17, então para cada 2
* 10 você pode obter o consecutivo trocando por 4 + 17, o que usa o
mesmo número de tijolos), o seguinte é 4 * 92 + 10 * 1 + 17 * 1. Isso
deve dar um trabalhão pra fazer na mão...

-- 
Bernardo Freitas Paulo da Costa

=
Instruções para entrar na lista, sair da lista e usar a lista em
http://www.mat.puc-rio.br/~obmlistas/obm-l.html
=


[obm-l] Re: [obm-l] Re: [obm-l] Re: [obm-l] RE: [obm-l] Contagem difícil

2012-09-30 Por tôpico Bernardo Freitas Paulo da Costa
2012/9/30 Pedro Júnior pedromatematic...@gmail.com:
 Olá Bernardo, você tem esse livro em pdf ou djavu?
Não, apenas no bom e velho papel mesmo.

 Ou sabes onde está postado para download?
Essas perguntas não cabem nesta lista.
-- 
Bernardo Freitas Paulo da Costa

=
Instruções para entrar na lista, sair da lista e usar a lista em
http://www.mat.puc-rio.br/~obmlistas/obm-l.html
=


[obm-l] Re: [obm-l] Re: [obm-l] Re: [obm-l] Re: [obm-l] Quantos dígitos tem o fatorial de 7000?

2012-09-26 Por tôpico Bernardo Freitas Paulo da Costa
2012/9/26 Bernardo Freitas Paulo da Costa bernardo...@gmail.com:
 2012/9/23 Rogerio Ponce abrlw...@gmail.com:
 Bem, 8 e' o ultimo digito diferente de zero em fatorial de 10.

 Alem disso, sabemos que
 8**1 termina em 8
 8**2 termina em 4
 8**3 termina em 2
 8**4 termina em 6
 8**5 termina em 8 novamente, estabelecendo um ciclo de 4 potencias ate' que
 o ultimo digito se repita novamente.

 Portanto, ao calcularmos o fatorial de 7000, partindo de 1, o que acontece
 e' que a cada 10 numeros (de 1 a 10, de 11 a 20, etc) o ultimo digito
 diferente de zero (no resultado) e' multiplicado por 8.
 Infelizmente, isso não é verdade. O maior problema mesmo é que isso de
 ser o último dígito não é muito estável por redução a aritmética
 modular. Fazendo umas continhas, eu descobri que:
 1*2*...*10 termina em 8 (como você calculou)
 11*12*...*20 termina em 8
 21*22*...*30 = 109027350432000, que termina em 2.
 31*32*...*40 = 3075990524006400
 41*..*50 = 37276043023296000
 51*...*60 = 273589847231500800
 61*...*70 = 1439561377475020800
 71*...*80 = 5974790569203456000
 81*...*90 = 20759078324729606400
 91*...*100 = 62815650955529472000
 101*...*110 = 170182143781102252800
 111*...*120 = 421188206644390348800
 121*...*130 = 96671689554375936
 131*...*140 = 2081693722421538086400
 141*...*150 = 4244078637389118528000

 E a seqüência é bem estranha: 8. 8. 2. 4. 6. 8. 8. 6, 4, 2, 8, 8, 6, 4, 8.
Só pra jogar fogo na situação: o produto
9765620*9765621*...*9765630 termina em 5.

-- 
Bernardo Freitas Paulo da Costa

=
Instruções para entrar na lista, sair da lista e usar a lista em
http://www.mat.puc-rio.br/~obmlistas/obm-l.html
=


Re: [obm-l] series telescopicas

2012-09-24 Por tôpico Bernardo Freitas Paulo da Costa
2012/9/24 Luís Lopes qed_te...@hotmail.com:
 Sauda,c~oes,

 Recebi o seguinte email:

 
 Boa tarde! Caro Prof. Luís Lopes, estou interessado em saber mais sobre as
 séries telescópicas, sendo mais especifico sobre a origem do termo
 telescópica, o porquê desse nome e como ele surgiu para definir esse tipo de
 série, desde já agradeço a atenção e estarei aguardando ansiosamente uma
 resposta. Obrigado!
 

 O porquê eu até acho que sei: aquele tubo da luneta se espicha e
 se recolhe sobre si mesmo. Daí a imagem com as séries telescópicas.

 Mas origem do termo, como e quando surgiu . ??
Bom, séries telescópicas devem ser coisa bem antiga. Provavelmente
até os gregos, por conta do paradoxo de Zenão, sabiam fazer coisas do
gênero. (escreva 1/2^n como 1/2^{n-1} - 1/2^n). O pessoal do tempo
pós-newtoniano também deve ter se divertido muito fazendo um monte de
contas com séries telescópicas porque isso tinha a ver com expansões
em série de equações diferenciais. Quanto à formalização do nome, com
certeza é depois do Galileu (e eu diria depois do Newton) porque
telescópios não existiam. Segundo
http://en.wikipedia.org/wiki/Talk%3ATelescoping_series#Gosper, parece
que um chute razoável seria entre o Weierstrass e o Hilbert, mas isso
é um trabalho bem grande de catar as origens.

Abraços,
-- 
Bernardo Freitas Paulo da Costa

=
Instruções para entrar na lista, sair da lista e usar a lista em
http://www.mat.puc-rio.br/~obmlistas/obm-l.html
=


[obm-l] Re: [obm-l] Característica de Euler para o Cone

2012-09-18 Por tôpico Bernardo Freitas Paulo da Costa
2012/9/18 Marcelo Gomes elementos@gmail.com:
 Olá pessoal da lista,

 Uma dúvida:

 1- A característica de Euler para sólidos varia de fórmula dependendo do
 sólido. Para os sólidos de Platão por exemplo vale a relação V + F = A + 2.
 Se for um toro (sólido com 1 furo) será uma outra fórmula. Depois pode-se
 falar sobre superfícies orientáveis e não orientáveis e etc.
Certo.

 2- Em relação ao cone corpo redondo ele tem quantas faces ? Qual a fórmula
 para ele ?
Hum, a definição habitual é amassar até ele virar um corpo
poliedral. Sem destruir furos e outras coisas, como você viu.

 3- Há alguma fórmula que generalize a relação de Euler (número de Euler)
 para qualquer sólido ? E superfície orientável e não orientável ?
Sim.

 Se alguém puder dar uma ajudinha, explicando esta parte, pois em minha
 pesquisa achei de forma bem rarefeita, que o cone possui uma face e uma
 base.
Eu não sei exatamente o que você está estudando agora, mas o quê você
precisa é estudar topologia algébrica. Procure por triangulações,
simplexos, complexos de cadeias (chain complex em inglês). E a fórmula
se chama característica de Euler-Poincaré.

 Abraços, Marcelo.

Abraços,
-- 
Bernardo Freitas Paulo da Costa

=
Instruções para entrar na lista, sair da lista e usar a lista em
http://www.mat.puc-rio.br/~obmlistas/obm-l.html
=


Re: [obm-l] probabilidade

2012-09-18 Por tôpico Bernardo Freitas Paulo da Costa
2012/9/18 Athos Couto athos...@hotmail.com:
 Provinha da UERJ?
 Hehe...

 20% acertaram porque sabiam.
Ok

 80% chutaram. Eram 4 alternativas e uma certa. 25% de chance de acertar.
Certo.

 Portanto, 0,8*0,25 = 0,2 = 20% acertaram chutando.
Hum, não sei não... marcar uma opção ao acaso não quer dizer que vai
ser isso. Veja bem, se você lançar um dado 6 vezes, não vai sair
necessariamente uma vez cada número. Claro que quanto mais vezes você
jogar, mais as proporções de cada número vão ficar próximas de 1/6
(lei dos grandes números) mas haverá também uma pequena oscilação
(proporcional à raiz quadrada do número de vezes que você jogar o
dado; Teorema central do limite). O que você fez vale, portanto, para
uma turma infinita (coitado do professor que corrigir as provas!). A
quantidade de alunos que acertou já é ela mesma uma variável aleatória
(Binomial, se eu não confundo os nomes), e a resposta depende (óbvio)
de cada valor possível.

Enfim, tudo depende do contexto do problema. Se você espera que o
sujeito seja um mínimo crítico quanto à contextualização, esse tipo de
enunciado mundo real é uma bela desgraça porque tá querendo dizer
uma coisa (os outros se dividem em 4 grupos de mesmo número e cada
grupo marcou uma das respostas) por uma via errada (marcar uma opção
ao acaso entre as 4) e esperando que o sujeito deduza o que era
para ser compreendido a partir de uma formulação que tem um sentido
completamente diferente. Matemáticamente falando, inclusive. E isso é
imperdoável. Contexto e mundo real é bom, mas adivinhação por ah,
isso é um problema de vestibular, então não pode estar querendo nada
muito complicado, então na verdade o que ele quer dizer é tal coisa é
apenas um entrave na educação.
-- 
Bernardo Freitas Paulo da Costa

=
Instruções para entrar na lista, sair da lista e usar a lista em
http://www.mat.puc-rio.br/~obmlistas/obm-l.html
=


[obm-l] Re: [obm-l] Quantos dígitos tem o fatorial de 7000?

2012-09-13 Por tôpico Bernardo Freitas Paulo da Costa
2012/9/13 ennius enn...@bol.com.br:
 Prezados Colegas,

 Qual o melhor método para calcular quantos dígitos tem o fatorial de 7000 
 (ou de qualquer outro número natural grande)?
Calcule o logaritmo em base 10.

Vai dar uma soma bem grande. A única coisa que falta é aproximar a
soma por uma integral, calculando o erro da aproximação.
-- 
Bernardo Freitas Paulo da Costa

=
Instruções para entrar na lista, sair da lista e usar a lista em
http://www.mat.puc-rio.br/~obmlistas/obm-l.html
=


Re: [obm-l] Ajuda em Polinomios

2012-09-12 Por tôpico Bernardo Freitas Paulo da Costa
2012/9/12 Rogerio Ponce abrlw...@gmail.com:
 Humm... eu justificaria da seguinte forma:

 Se o polinomio resto da divisao de P(x)/Q(x) assume o valor zero para
 infinitos valores de x, ou ele possui uma quantidade infinita de raizes ou
 ele e' identicamente igual a zero.
 Como ele nao pode ter uma quantidade infinita de raizes, entao ele e' nulo.
 Portanto Q(x) divide P(x).

 Isso seria suficiente?

 []'s
 Rogerio Ponce
Acho que não, porque P(x)/Q(x) ser inteiro não implica que o resto é
zero. Veja que (x^2 + 1)/(x + 1) tem resto 2 (e quociente = x - 1),
mas em x=1 temos que P(x)=2=Q(x). Acho que tem que ter algum argumento
de limpeza como fez o Ralph.

 2012/9/12 Heitor Bueno Ponchio Xavier heitor.iyp...@gmail.com

 Não consigo fazer a seguinte questão:
 Mostre que se P(x) e Q(x) são polinômios de coeficientes inteiros tais
 que P(x)/Q(x) é inteiro para infinitos valores inteiros de x então Q(x)
 divide P(x).






-- 
Bernardo Freitas Paulo da Costa

=
Instruções para entrar na lista, sair da lista e usar a lista em
http://www.mat.puc-rio.br/~obmlistas/obm-l.html
=


[obm-l] Re: [obm-l] Função raiz enésima é crescente

2012-09-03 Por tôpico Bernardo Freitas Paulo da Costa
2012/9/3 Paulo Argolo pauloarg...@outlook.com:
 Caros Colegas,

 Como podemos provar que é crescente a função f(x) = x^(1/n)?

 (x é número real positivo, n é número natural diferente de zero.)
Tudo depende de como você define essa função... Principalmente para os
números reais. E quanto rigor também você quer na demonstração ;-)

Abraços,
-- 
Bernardo Freitas Paulo da Costa

=
Instruções para entrar na lista, sair da lista e usar a lista em
http://www.mat.puc-rio.br/~obmlistas/obm-l.html
=


Re: [obm-l] Integral

2012-08-30 Por tôpico Bernardo Freitas Paulo da Costa
2012/8/30 Samuel Wainer sswai...@hotmail.com:
 Me pediram um exemplo de uma função que tem integral finita, mas que f^2 não
 tem integral finita.

 Fiquei quebrando a cabeça um tempão, mas não consegui.

 Alguém tem alguma ideia?

Tem vários exemplos clássicos, mas o importante é *como* fazer.

Existem dois jeitos de uma integral ser infinita: porque o domínio é
grande, ou porque o valor é grande. Refrescando a memória, você deve
lembrar que 1/x tem *ambos* problemas. Quando você integra de 1 até
infinito, dá infinito (log(M) - log(1) com M - infinito) e quando
você integra de 0 até 1 também (log(1) - log(eps) com eps - 0). Bom,
temos um candidato (se der!) para f^2.

Agora, vejamos. Tirando a raiz quadrada de 1/x, quando x - infinito,
isso quer dizer que a função fica MAIOR AINDA! Portanto, a integral
com certeza ainda é infinita. Aliás, qualquer função cuja integral dá
+infinito sem divergir em algum ponto (isso incluiu os +- infinito),
ou é maior do que 1 num intervalo de tamanho infinito (e isso explica
porque que dá +infinito a integral) e nesse caso não adianta tirar
raiz, vai continuar  1 ; ou então é menor do que 1, e quando você
tira a raiz, fica maior ainda. Assim, nunca vai dar certo no
infinito.

Sobrou o caso de ser na parte (0,1). Aqui, como  x  1, temos que 1/x
 raiz(1/x)  1, ou seja, a função DIMINUIU. Isso é muito bom, porque
(se você lembra) a 1/x é o limite de 1/x^alfa ter integral finita ou
não. Agora, basta verificar que realmente 1/raiz(x) é integrável em
(0,1). Isso eu deixo pra você conferir (mas a gente acabou de provar
que ela NÃO é integrável em (1, infinito), ou seja, ainda falta um
pouquinho).

A última parte é uma roubadinha  (ou roubadona, para o pessoal
analítico como Cauchy, Euler e amigos): pegue a função raiz(1/|x|)
para x entre -1 e 1, e depois cole uma função afim qualquer que
ligue até o zero, por exemplo subindo de -2 até -1 numa reta, depois
seguindo a 1/raiz(x), sobe, vai ao infinito no zero, volta até 1 em
x=1, e depois desce numa reta simétrica até o zero em x=2. Pronto,
essa função é com certeza integrável, porque é a soma de duas que são,
mas o quadrado dela tem uma parte que vai dar infinito.


-- 
Bernardo Freitas Paulo da Costa

=
Instruções para entrar na lista, sair da lista e usar a lista em
http://www.mat.puc-rio.br/~obmlistas/obm-l.html
=


Re: [obm-l] Mais divisibilidade

2012-08-23 Por tôpico Bernardo Freitas Paulo da Costa
2012/8/22 João Maldonado joao_maldona...@hotmail.com:
 Suponha que  vale para n

 Logo 10^(3n)-1 = k.3^(n+2)

 10^(3n+3)-1000 = 1000k3^(n+2)

 10^(3n+3)-1 = 1000.k.3^(n+2) + 999

 Analizemos 1000.k.3^(n+2) + 999 modulo 3^(n+3)

 1000.k.3^(n+2) + 999 modulo 3^(n+3) = 333.k.3^(n+3) + k.3^(n+2) + 999
 Vemos claramente que como a maior potência de 3 que divide 999 é 3, logo
 temos que a expressão não vale isso não vale para n=2

 É fácil ver que (10^6-1)/(3^4)  não é inteiro

 Talvez a expressão esteja escrita errada não?

 Eu interpretei como 10^(3n)-1
 Talvez seja 1000n-1
 ou até 10^(3n-1)

 Mesmo assim, nenhuma vale
 n=2, 1999 não divide 3
 n=1, 100 não divide 3

 Talvez você tenha errado na digitação ou algo assim
 Tem certeza que o exercício é esse?
Eu chuto que seja 10^(3^n) para voce fatorar x^3 - 1 = (x-1)(x^2 + x + 1)

Abraços,
-- 
Bernardo Freitas Paulo da Costa

=
Instruções para entrar na lista, sair da lista e usar a lista em
http://www.mat.puc-rio.br/~obmlistas/obm-l.html
=


Re: [obm-l] divisibilidade(3)

2012-08-21 Por tôpico Bernardo Freitas Paulo da Costa
2012/8/21 marcone augusto araújo borges marconeborge...@hotmail.com:
 Mostre,para todo n E N,que

 notação: a exp b significa´ a elevado a b´
 a² -a + 1 divide a exp (2n+1) + (a-1) exp (n+2)
Recorrencia!

Mostre que vale para n=0 (facil!) e depois use que
x | cx + d = x | d
para simplificar (voce vai ter que somar e subtrair termos iguais para
poder fatorar o a^2 - a + 1.

Abracos,
-- 
Bernardo Freitas Paulo da Costa

=
Instruções para entrar na lista, sair da lista e usar a lista em
http://www.mat.puc-rio.br/~obmlistas/obm-l.html
=


Re: [obm-l] Divisibilidade(2)

2012-08-16 Por tôpico Bernardo Freitas Paulo da Costa
2012/8/16 João Maldonado joao_maldona...@hotmail.com:
 (a³+4)/(a-2) = (a³-8+12)/(a-2) = (a²+2a+4) + 12/(a-2) = 12 tem que ser
 divisível por a-2 - a=3, 4, 5, 6, 8, 14
 (a³-3)/(a+3) = (a³+27-30)/(a+3) = (a³-3a+9) -30/(a+3) - 30 tem que ser
 divisível por a+3 - a=0, 1, 2, 3, 7, 12
Nao esqueca que -1 divide 12, portanto a-2 = -1 = a = 1 tambem vai
servir. E as outras solucoes tambem, eh claro.
-- 
Bernardo Freitas Paulo da Costa

=
Instruções para entrar na lista, sair da lista e usar a lista em
http://www.mat.puc-rio.br/~obmlistas/obm-l.html
=


Re: [obm-l] MQ=MA=MG=MH

2012-06-15 Por tôpico Bernardo Freitas Paulo da Costa
2012/6/15 Carlos Nehab carlos.ne...@gmail.com:
 Oi, Felippe,

 Se o seu enunciado é:
 Dentre os ternos (x, y, z) , com x, y e z reais, que satisfazem a x+y+z=5 e
 xy+yz+xz=3 calcule o maior valor possível para x,
 então eu achei outro resultado:
 Usando (x+y+z)^2 = x^2+y^2+z^2 + 2(xy+yz+zx) obtemos
 x^2+y^2+z^2= 19
 Logo, o maior valor de x é raiz(19) que é maior que o 13/3 (e os
 correspondentes valores de y e z são 0).
Cuidado, Nehab, porque x=raiz(19), y=z=0 não satisfaz x+y+z=5 (que é
racional) e, pior ainda, xy + yz + zx = 0, e não 3. Isso (mais uma
vez) serve como cota superior, mas não garante a existência da
solução.

Se der tempo (duvido...) eu mando uma na força bruta por
multiplicadores de Lagrange, que é sem dúvida mais geral do que a do
Ralph, e pode dar uma iluminada. (aliás, esse problema parece bastante
com outro dessa semana...)

abraços,
-- 
Bernardo Freitas Paulo da Costa

=
Instruções para entrar na lista, sair da lista e usar a lista em
http://www.mat.puc-rio.br/~obmlistas/obm-l.html
=


[obm-l] Re: [obm-l] RE: [obm-l] Colômbia- álgebra

2012-06-10 Por tôpico Bernardo Freitas Paulo da Costa
 Sejam a,b,c reais tais que
 a^12+b^12+c^12=8
 [(a-b)^2+(b-c)^2+(c-a)^2]/abc= 6/(a+b+c)

 Calcule a^6+b^6+c^6.

2012/6/10 Thiago Tarraf Varella thiago_...@hotmail.com:
 mas desenvolvendo a 2a. equação na raça, obtém-se
 a³ + b³ + c³ = 6abc

 Isso deve ajudar... alguma sugestão do que fazer agora?

Isso dá uma boa ajuda, mas acho que a conclusão final é que não dá
para determinar (numericamente) a^6 + b^6 + c^6.

Eu argumentaria assim: seja P(x) = x^3 - Ax^2 + Bx - C um polinômio
cujas raízes são a, b e c. Daí, como todas as quantidades são
homogêneas, vamos gastar braço e escrever tudo em função de A, B e C.
Vai dar um monte de trabalho, mas azar. Daí, você obtém uma fórmula
feia para S_12 e outra para S_6, as somas de potências (essa até é
bonitinha:

3 * S_6 = A^4 (A² - 3B) - 3B^3,

se eu não me enganei.)

Bom, a fórmula para S_12 é feia, mas não morde. Dá uma equação de grau
6 homogênea em A^2 e B, fixe um, ache o outro, dado que S_12 = 8.

Bom, agora vem um argumento meio abstrato.

Note que a região (S_12 = 8 inter S_3 = 6C) é de dimensão 1 em C^3 (e
também em R^3, a menos que sejam pontos isolados, mas não é o caso), e
por ser de dimensão 1, a gente chama de curva. Note que ela pode ter
várias partes (componentes conexas), mas isso não importa. Note também
que o valor que a gente quer calcular é S_6, que também é homogêneo.
Então há duas possibilidades: ou ele é constante nessa curva de
dimensão 1 (e basta pegar uma solução qualquer a, b, c e ver o quanto
dá), ou ela é não-constante em *cada componente da curva*, e (em C^3)
assume *todos* os valores complexos possíveis em cada componente
conexa. O mais legal é que essa dicotomia continua mais ou menos
válida em R (que é o que nos interessa!), e o comportamento em R é o
mesmo que o de C: se for constante em C, será em R (óbvio!), e a
recíproca também vale.

Bom, daí você apela para qualquer software de cálculo algébrico, bota
a equação, chuta uns valores para B (ou A, tanto faz), pede pra
resolver a equação do S_12, substitui no S_6, e vê se dá igual. Se eu
não fiz nenhuma besteira no caminho, existem valores de A e B para os
quais dá diferente. Note que os valores de A e B que eu escolhi, muito
provavelmente vão dar uma equação em que um de a, b e c será complexo.
Mas isso não importa, pelo que eu disse antes: se existir alguma forma
de S_6 mudar, mesmo que passando pelos complexos, também será o caso
se a, b e c forem apenas reais. Ufa!

Abraços,
-- 
Bernardo Freitas Paulo da Costa

=
Instruções para entrar na lista, sair da lista e usar a lista em
http://www.mat.puc-rio.br/~obmlistas/obm-l.html
=


Re: [obm-l] Prova combinatoria

2012-06-06 Por tôpico Bernardo Freitas Paulo da Costa
2012/6/6 marcone augusto araújo borges marconeborge...@hotmail.com:
 notação:(n,p)--número binomial de numerador n e denominador p

 1 + 2(n,1) + 4(n,2) + ...[2^(n-1)](n,n-1)+ [2 ^n](n,n) = 3^n
 Se desenvolvermos (x + 2y)^n e substituirmos x por 1 e y por 1,encontraremos
 a expressão do lado esquerdo,que será igual a (1+ 2)^n
Veja que isso é também a expansão de (x + y)^n com x=1 e y=2.

 O exercício pede para encontrar uma prova combinatória.
Uhm, pra tentar uma prova combinatória, eu faria *antes de mais nada*
uma prova combinatória da mesma fórmula só que com x=1 e y=1. Talvez
você até já conheça uma. Daí tente generalizar!

 Já pensei,pensei e não saiu.

Abraços,
-- 
Bernardo Freitas Paulo da Costa

=
Instruções para entrar na lista, sair da lista e usar a lista em
http://www.mat.puc-rio.br/~obmlistas/obm-l.html
=


Re: [obm-l] Densidade

2012-05-28 Por tôpico Bernardo Freitas Paulo da Costa
2012/5/28 João Grillo matli...@hotmail.com:
 Seleciona-se, ao acaso, um ponto do quadrado unitário {(x,y): 0 = x = 1, 0
 = y = 1}. sejam X e Y as coordenadas do ponto selecionado.

 Qual a densidade conjunta de X e Y?
Esse problema é um problema.

Me explico: quando se diz ao acaso, aleatoriamente, ..., isso
supõe uma distribuição de probabilidade a partir da qual se pode
efetuar uma ação ao acaso. Sem dar uma distribuição de
probabilidade, nada feito. Assim, a pergunta do seu problema pede na
verdade é um elemento fundamental do enunciado. Mais ou menos como
Mario tinha 20 bananas, e comeu todas elas. Quantos quilos ele engordou?
A resposta depende (óbvio) do peso das bananas. Você pode supor que
cada banana pesa 350g, mas isso não foi dito. Poderia ser 100 (uma
banana nanica) ou 500 (sei lá).

Assim, esse tipo de questão não faz muito sentido...

Abraços,
-- 
Bernardo Freitas Paulo da Costa

=
Instruções para entrar na lista, sair da lista e usar a lista em
http://www.mat.puc-rio.br/~obmlistas/obm-l.html
=


Re: [obm-l] Eu preciso de ajuda com este problema

2012-05-19 Por tôpico Bernardo Freitas Paulo da Costa
2012/5/19 João Maldonado joao_maldona...@hotmail.com:
 Cara, se voce traduzir isso que voce quis dizer ai a gente ate pode te
 ajudar
Acho que o problema vem justamente de traduzir. Se eu não me engano, o
drechum deve ter usado um tradutor automático para português, e por
isso que o haver foi conjugado no plural em vez de continuar no
singular como manda a nossa gramática.

Drechum: você é Cubano como sugere o seu e-mail? Eu suspeito que você
esteja usando uma tradução inglês - português, talvez seja muito mais
simples usar uma tradução espanhol - português, ou (melhor ainda)
tentar escrever diretamente em português, que é muito, muito, muito
próximo do espanhol.

 From: drec...@prodind.gecpri.co.cu
 To: obm-l@mat.puc-rio.br
 Subject: [obm-l] Eu preciso de ajuda com este problema
 Date: Sun, 20 May 2012 10:19:45 -0400

 Em uma reunião há 12 pessoas. É conhecido que para cada duas pessoas A e B
 da reunião é (pelo menos) outra pessoa C da reunião da que é o amigo A e de
 B. Determinar o número mínimo até mesmo de de amigos que hão na reunião.
 Cada pessoa pode integrar vários pares. Se X é então o amigo de Y que Y é o
 amigo de X

Se eu entendi o que quer dizer o ńumero mínimo de amigos (mas
confesso que não é claro), isso talvez queira dizer o ńumero mínimo
de arestas do grafo de amizade. Se a amizade é simétrica (como você
disse) mas não transitiva, eu acho que o mínimo de arestas é 11 + 6,
construído da seguinte forma:

Escolha uma pessoa (o anfitrião) que conhece todo mundo, e que todo
mundo conhece. Em seguida, divida os 11 restantes em 5 pares e uma
pessoa isolada. As pessoas de cada par, além de conhecerem o
anfitrião, se conhecem. Enfim, a pessoa isolada conhece alguém de
outro grupo.

Mas tem que provar que essa é a solução ótima...

Abraços,
-- 
Bernardo Freitas Paulo da Costa

=
Instru��es para entrar na lista, sair da lista e usar a lista em
http://www.mat.puc-rio.br/~obmlistas/obm-l.html
=


[obm-l] Re: [obm-l] dúvida em teoria dos números

2012-05-17 Por tôpico Bernardo Freitas Paulo da Costa
On Thu, May 17, 2012 at 12:18 PM, Marco Antonio Leal
marcoantonio_elemen...@hotmail.com wrote:
 Provar que 10 ^11 - 1 é divisivel por 100
Um número só com noves não tem grandes chances de ser divisível por
100. Pior ainda, um número ímpar não é divisível por nenhum par. E não
adianta nem tentar mudar o sistema de numeração (base 7, por
exemplo...) porque 100 = b^2 que divide b^11, mas não divide 1, logo
não divide a diferença.

Talvez seja 11^10 - 1? Se for o caso, binômio de Newton nele!
11 = 10 + 1, 11^10 = (10 + 1)^10 = 10^10 + ... + 10*9/2 * 10^2 * 1^8 +
10 * 10^1 * 1^9 + 1^10 = 100*(número feio, mas inteiro) + 10*10*1 + 1
= 100(número feio + 1) + 1.

Abraços,
-- 
Bernardo Freitas Paulo da Costa

=
Instruções para entrar na lista, sair da lista e usar a lista em
http://www.mat.puc-rio.br/~obmlistas/obm-l.html
=


Re: [obm-l] Divisibilidade

2012-05-11 Por tôpico Bernardo Freitas Paulo da Costa
2012/5/11 Thiago Bersch thiago_t...@hotmail.com
 Mostre que se 19 | 3x + 7y então 19 | 43x + 75y
Oi Thiago,

todos esses problemas de divisibilidades mágicas usam duas coisas:
- a | a * b para todo b inteiro
- Se a | X, então ( a | Y = a | X+Y )

Note que essa última implicação pode (e deve) ser usada com números
negativos. Assim, se X = p + q, você pode usar Y = -q para deduzir que
a | p. Daí, é só achar um jeito de ter a | -q, do mesmo jeito que no
problema do 13 divide

Bons estudos,
--
Bernardo Freitas Paulo da Costa

=
Instruções para entrar na lista, sair da lista e usar a lista em
http://www.mat.puc-rio.br/~obmlistas/obm-l.html
=


[obm-l] Re: [obm-l] dúvida em uma afirmação de um vestibular da UEM

2012-05-01 Por tôpico Bernardo Freitas Paulo da Costa
2012/5/1 Vanderlei * vanderma...@gmail.com:
 Pessoal, a afirmação a seguir é verdadeira ou falsa? Penso que seja
 verdadeira, porém o gabarito do vestibular diz ser falsa! Preciso de ajuda!

 Em um plano, existem duas figuras F1  e F2, cujas bases estão sobre uma reta
 r  do plano, com a seguinte propriedade:
 toda reta paralela à reta  r que intersecta F1  e F2  determina segmentos
 em  F1  e em  F2  de mesma medida .
 Então, a área de  F1 é igual à área de  F2.
Talvez seja uma sutileza do enunciado. O que é verdade é
Se toda reta r determina segmentos de comprimentos iguais em F1 e F2,
então F1 e F2 têm a mesma área

O que poderia acontecer no caso do enunciado (levando *muito* ao pé da
letra, e francamente o tipo de questão que me deixa desgostoso com o
modo como as pessoas encaram a matemática) seria um triângulo
equilátero para F1 e dois triângulos equiláteros (em forma de
ampulheta) para F2, tocando-se por um vértice. Assim, toda reta que
encontra F1 *E* F2, determina segmentos iguais, mas algumas retas
encontram só F2 (e outras poderiam encontrar só F1... ninguém falou
que as figuras eram conexas...) e a diferença de áreas vem daí.

Esse tipo de questão, numa múltipla escolha, me parece pouco
apropriado. A mesma questão, com um enunciado totalmente claro (e não
com tantas ambigüidades possíveis) e que pedisse uma demonstração ou
um contra-exemplo (e não apenas Verdadeiro ou Falso), talvez fosse
melhor. Mas continuo achando que usar isso para determinar a qualidade
dos candidatos é uma deformação do real propósito da matemática. Não
se está medido a compreensão de um assunto, mas a pura atenção ao
mínimo detalhe.

Faz sentido usar isso em sala de aula: ainda mais para explicar a
necessidade de dar um enunciado *sempre* completo na hora da verdade
(porque é a exigência deontológica da disciplina), mas também para
dizer olha, a gente quer que tal coisa seja verdade, mais ou menos
assim, e a partir daí construir o tal do enunciado completo, que não
cai do céu.

Abraços,
-- 
Bernardo Freitas Paulo da Costa

=
Instruções para entrar na lista, sair da lista e usar a lista em
http://www.mat.puc-rio.br/~obmlistas/obm-l.html
=


Re: [obm-l] Divisibilidade

2012-04-26 Por tôpico Bernardo Freitas Paulo da Costa
2012/4/26 marcone augusto araújo borges marconeborge...@hotmail.com:
 Prove que, entre 2^(n+1) números naturais quaisquer,existem 2^n números cuja
 soma é divisível por 2^n

 Eu sei que em uma divisão por 2^n existem 2^n  restos possíveis
 Se em 2^n divisões ocorressem 2^n restos iguais a r,a soma deles seria
 r*2^n,que é divisível por 2^n
O problema dessa idéia é que você não tem certeza que dá pra fazer de
forma independente...

 Não sei se conseguiria resolver por congruência,mas eu gostaria de ver uma
 solução por outro caminho.
Bom, olhando a questão, parece ser um caso de recorrência. E é mesmo!
(enfim, funciona)

Mostre que é verdade para n = 1. Esse caso é fácil, mas já é a base de tudo...
Agora, tente ver como faz para n = 2. Você tem 8 números
(quaisquer!!!) e você tem que conseguir 4 cuja soma seja divisível por
4. Por indução, você sabe que para cada decomposição 8 = 4+4, você
consegue 2 vezes 2 números cuja soma é divisível por 2. Mas isso não
garante que é divisível por 4!! Podia dar 2 + 0... e aí? A dica é ver
que o caso n = 1 não é optimal...

Abraços,
-- 
Bernardo Freitas Paulo da Costa

=
Instruções para entrar na lista, sair da lista e usar a lista em
http://www.mat.puc-rio.br/~obmlistas/obm-l.html
=


Re: [obm-l] Fibonacci

2012-04-25 Por tôpico Bernardo Freitas Paulo da Costa
2012/4/25 Carlos Nehab carlos.ne...@gmail.com:
 Oi Marcone,

 A forma mais simples de provar esta joça é usar duas coisinhas:

 1) a^(km) - b^(km) é divisível por a^m - b^m   e
 2) a formuleta do Binet para o termo geral da sequência de Fibonacci...
The chato Strikes Back

Claro que o raiz(5) do denominador vai pros dois F_(km) e F_m. O
problema é que o outro fator, a saber

a^{(k-1)m} + a^{(k-2)m} * b + ... + b^{(k-1)m}

tem um montão de números irracionais (começando com a e b,
respectivamente (raiz(5) - 1)/2 e (raiz(5) + 1)/2, que eu chamo de
phi-zinho e Phi-zão). Claro que F_(km) e F_m são inteiros (não é ?) e
portanto o outro fator é, apesar dos pesares (irracionais),
racional. Então, ainda tem um pouquinho de trabalho (e talvez um
montão...) pra provar que essa galera toda somada dá um número
inteiro.

Abraços,
-- 
Bernardo Freitas Paulo da Costa

=
Instruções para entrar na lista, sair da lista e usar a lista em
http://www.mat.puc-rio.br/~obmlistas/obm-l.html
=


Re: [obm-l] INTEGRAL COMPLEXA

2012-04-23 Por tôpico Bernardo Freitas Paulo da Costa
2012/4/23 Rogério Possi Júnior roposs...@hotmail.com:
 Pessoal,

 Segue uma questão de integral complexa:

 INTEGRAL DE LINHA [(1 / ( (Z^100 + 1).(Z-4) )]dZ, onde a integral é
 calculada sobre C: MÓD[Z]=3

Você já ouviu falar de resíduos? Daonde surgiu esse problema?

Abraços,
-- 
Bernardo Freitas Paulo da Costa

=
Instruções para entrar na lista, sair da lista e usar a lista em
http://www.mat.puc-rio.br/~obmlistas/obm-l.html
=


[obm-l] Re: [obm-l] aritmética

2012-04-16 Por tôpico Bernardo Freitas Paulo da Costa
2012/4/16 Carlos Nehab carlos.ne...@gmail.com:
 2) Solução
 X = 0,3737...  Y = 0,7373...
 Na primeira base r1:
 (r1^2-1).X = 3r1+7
 (r1^2-1).Y = 7r1+3
 Somando, (r1^2-1)(X+Y) = 10(r1+1), ou seja,
 (r1-1)(X+Y)=10    (A)
 Dai já sabemos que r1-1 = 1, 2, 5 ou 10. Mas r1  7, logo r1 = 11 e X + Y =
 1
Hum, X e Y são frações, certo? Porque então você insiste que X+Y seja
inteiro (única razão que eu vi para que r1 - 1 divida 10) ? Nesse caso
até que dá certo, mas sei lá, podia ser que X+Y = 1/2 (mas teria
talvez que mudar a expressão na base r2)

Abraços,
-- 
Bernardo Freitas Paulo da Costa

=
Instruções para entrar na lista, sair da lista e usar a lista em
http://www.mat.puc-rio.br/~obmlistas/obm-l.html
=


[obm-l] Re: [obm-l] Re: [obm-l] sugestão de material

2012-04-09 Por tôpico Bernardo Freitas Paulo da Costa
 2012/4/8 Marcelo Costa mat.mo...@gmail.com:
 Devo fazer minha prova de língua estrangeira, optei por francês, será que
 algum dos srs. poderia me enviar alguns resumé de trabalhos  na área de
 matemática para que eu possa treinar, ou mesmo me apontar algum link?
Um site fantástico com uma quantidade de matemática impressionante e
muito bem escrita é o Images des maths :
http://images.math.cnrs.fr/, que toda pessoa sabendo ler em francês
deveria conhecer!

Abraços,
-- 
Bernardo Freitas Paulo da Costa

=
Instruções para entrar na lista, sair da lista e usar a lista em
http://www.mat.puc-rio.br/~obmlistas/obm-l.html
=


Re: [obm-l] Fibonacci

2012-04-08 Por tôpico Bernardo Freitas Paulo da Costa
2012/4/8 Rogerio Ponce abrlw...@gmail.com:
 Ola'  Gabriel,
 se cada casal viver por k+0.5 meses (0.5 e' para nao haver confusao
 sobre a geracao de descendentes no momento em que o casal morre),
 entao basta voce subtrair a quantidade de coelhos com idade igual ou
 mais velhos que k+1 meses.
 Assim, a resposta para o seu problema seria
 F(n) - F(n-k-1)
Bom, vou dizer que eu achei estranha essa resposta porque a sua
seqüência satisfaz a mesma recorrência que o problema original,
enquanto que eu acho que a recorrência aqui é

G(n+k+2) = G(n+k+1) + G(n+k) - G(n)

(Ou seja, entre o mês n+k+1 e o seguinte, os coelhos que têm mais de 1
mês geram um novo casal, o que corresponde ao termo + G(n+k), e os
que são bem velhinhos morrem, o que dá o termo - G(n) ; isso dá
k+1+0.5 meses de vida para os coelhos, seguindo a sua idéia).

Ora,

F(n+k+2) - F(n+alfa+2) = F(n+k+1) + F(n+k) - (F(n+alfa+1) + F(n+alfa))
= [F(n+k+1) - F(n+alfa+1)] + [F(n+k) - F(n+alfa)]

que é a recorrência de Fibonacci (claro) para X(n) = F(n+k) -
F(n+alfa). A mesma demonstração diz que nenhuma combinação de F(n)'s
pode ser solução da recorrência modificada.

Daí eu acho que você esqueceu de subtrair também os filhos que F(n)
inclui para os coelhos velhos demais.

Quanto à recorrência que eu propus acima, eu acho que ela é bem mais
chata de resolver porque o polinômio característico depende de k;
x^(k+2) = x^(k+1) + x^k - 1.

Sem computador, você já pode perceber que há sempre uma solução x = 1.
Se k for ímpar, você também tem x = -1. Também sem computador, você
pode acreditar que a maior solução é sempre menor do que Phi = (1 +
raiz(5))/2, porque tem que haver menos coelhos do que no caso que eles
são imortais, e você pode até chutar que a maior solução é
crescente. Com um computador, você pode continuar chutando: as
raízes reais são apenas as que eu mostrei acima, e as outras são
complexas de módulo menor do que 1. Talvez dê pra provar isso sem
muito trabalho, mas sei lá. Eu acho que a gente também pode chutar que
o módulo delas é maior do que o módulo de phi = (1 - raiz(5))/2, mas
eu não tenho grandes justificativas pra isso não. Também parece que o
módulo delas tende a 1 (ou seja de todas as raízes exceto a maior), e
talvez isso seja mais fácli de demonstrar.

Ah, outra coisa importante de chutar (depois disso tudo) é que as
raízes são todas simples, porque daí basta saber qual é a maior, e o
coeficiente das raízes 1 e -1 para achar os valores para n grande por
truncamento ;)

Abraços,
-- 
Bernardo Freitas Paulo da Costa

=
Instruções para entrar na lista, sair da lista e usar a lista em
http://www.mat.puc-rio.br/~obmlistas/obm-l.html
=


Re: [obm-l] Fibonacci

2012-04-08 Por tôpico Bernardo Freitas Paulo da Costa
2012/4/8 Gabriel Guedes g.a.gue...@gmail.com:
 Oi amigos da lista.
 Bernardo, mas ai estaria implícito nas suas hipóteses que a quantidade dos
 que morrem é igual as do que nasceram a certo tempo atrás.
Ué, não foi isso que você quis dizer com morrem após um determinado
período de tempo?

Eu interpretei dizendo: os coelhos nascidos há k + 1 meses morrem. No
tempo n+k+2, os que nasceram há k+1 meses foram os que nasceram no
tempo n+1, e disse que valia G(n), porque eu não prestei atenção.
Primeiro, deveria ser somente G(n-1), porque os do tempo G(n) contém
também os que acabaram de nascer e que portanto não geraram filho
nenhum.

Mas o erro mais sério (como eu acho que você percebeu) é que os
coelhos que estavam vivos no tempo G(n-1) talvez já tenham morrido no
tempo G(n) e portanto não teriam tido tempo de fazer um filho para o
tempo n+1.

  Acredito que
 deveriam existir três relações F para os nascimentos ( que é a seq de
 Fibonacci que conhecemos). G uma outra para a morte dos coelhos. E uma H em
 função de F e G para modelar o novo problema.
 O que acha?
Eu acho que é por aí, mas eu não tenho certeza que F(n) aparecerá.

Como eu disse aí em cima, o chato é você saber quantos nasceram no
tempo n. Chamemos esta relação de N(n) para nascimentos. Os que
morrem no tempo (n+k) são exatamente os que nasceram no tempo n. Seja
C(n) o número de coelhos total. Vamos tentar fazer uma relação de
recorrência.

C(n+k+1) = C(n+k) + N(n+k) - N(n)
- no tempo n+k+1, nascem e morrem alguns coelhos, cuja esperança de
vida é k meses.

O número de nascimentos no tempo n+k é igual ao número de coelhos, do
tempo n+k, em idade de reproduzir. Ou seja, os coelhos nascidos há
mais de um mês, e há menos de k meses. Os nascidos há mais de um mês
são C(n+k-1). Dentre estes, os que nasceram há k meses, ou seja
N(n-1), morreram. Portanto N(n+k) = C(n+k-1) - N(n-1).

Isso dá um sistema de recorrências que deve dar pra resolver. Se eu
não me enganei nas contas,

N(n+k+2) = N(n+k+1) + N(n+k) - N(n+1)

que (olhe só que a coincidência!) é a primeira recorrência que eu
tinha indicado. Porque realmente eu estava mais pensando em
nascimentos do que em totais. Veja que faz sentido: os que nascem no
tempo n+k+2 são os que nasceram antes, mais os que vão poder nascer
dado ao envelhecimento dos mais jovens (que nasceram há exatamente 2
meses atrás, e portanto não haviam contribuído a N(n+k+1), mas agora
vão começar a se multiplicar) menos os que acabaram de bater as botas.
Daí para calcular C(n) é um pulinho, pela segunda equação.

Bom problema! Mas acho que pela dificuldade prática de cálculo (eu
continuo acreditando em uma solução diferente por cada k diferente) é
menos elementar do que o Fibonacci original.

Abraços(n+k+3),
-- 
Bernardo Freitas Paulo da Costa

=
Instruções para entrar na lista, sair da lista e usar a lista em
http://www.mat.puc-rio.br/~obmlistas/obm-l.html
=


[obm-l] Re: [obm-l] RE: [obm-l] insegurança

2012-04-06 Por tôpico Bernardo Freitas Paulo da Costa
2012/4/6 João Maldonado joao_maldona...@hotmail.com:
 O principio esta certo, mas se for uma prova dissertativa, talvez eles nao
 te dessem nota por nao estar tao completa essa inducao

 Eu escreveria assim

 Tese f(ak) é PA de razao f(r) para k inteir =n
 Condicao inicial f(a1)=f(a)+f(r)
 Hipotese f(k)=f(k-1)+f(r) qualquer que seja k inteiro menor que n
 Prova f(an)=f(an-1+r)=f(an-1)+f(r)
Isso não é uma indução, é uma prova direta (como a original,
inclusive). Veja que você não usa que f(a_k) = f(a_{k-1}) + f(r) para
deduzir a mesma coisa para f(a_{k+1}). A única propriedade necessária
é a linearidade mesmo. Eu prefiro a forma original, inclusive:
simples, direta, clara. E se eu estivesse num dia ruim, eu tiraria
pontos da sua, porque ficaria com a impressão que o aluno está
tentando me enrolar.

-- 
Bernardo Freitas Paulo da Costa

=
Instruções para entrar na lista, sair da lista e usar a lista em
http://www.mat.puc-rio.br/~obmlistas/obm-l.html
=


[obm-l] Re: [obm-l] RE: [obm-l] Re: [obm-l] RE: [obm-l] insegurança

2012-04-06 Por tôpico Bernardo Freitas Paulo da Costa
 ponto, faça como quiser, cada um tem a
 sua mentalidade
Cara, se você insistir, eu tiro ponto até do seu professor por ter te
obrigado a botar indução onde ela não foi chamada!

Abraços,
-- 
Bernardo Freitas Paulo da Costa

=
Instruções para entrar na lista, sair da lista e usar a lista em
http://www.mat.puc-rio.br/~obmlistas/obm-l.html
=


<    1   2   3   4   5   6   7   >